You are on page 1of 105

SOAL-SOAL LITERASI BAHASA INGGRIS

Question No. 1 is based this following text


Isotonic drinks or drinks for sports are also called sports drinks. Sports drinks are very
beneficial, especially if you often exercise at a high level (3-5 times per week for about 45-90
minutes). Sport drinks are divided into several groups, but the goal is almost the same for all,
namely to restore stamina after exercising. Sports drinks can also increase the speed of
recovery after exercise so that we can exercise better for a long time. Usually used to
maintain body weight or fitness, muscle strength, better body appearance, and body fitness.

1. The discourse above contains about...


A. Isotonic drinks or sports drinks and their functions
B. Isotonic drinks to maintain the body
C. Isotonic drinks as a substitute for water
D. Isotonic drinks for fitness
E. Isotonic drinks for daily consumption

Jawaban: A
Pembahasan:
Wacana pada soal membahas tentang minuman isotonik atau sport drink dan
fungsinya bagi tubuh. Maka, pilihan jawaban yang tepat adalah A.

Question No. 2 is based this following text.


El James' Fifty Shades of Gray novel sold over 100 million copies of the James Bond series
written by Ian Fleming. The novel has been sold worldwide. However, in terms of the
number of book sales, the novel still lost to the best-selling Twilight book series written by
Stephanie Meyer and The Da Vinci Code by Dan Brown. Twilight novel sales exceeded 200
million copies.

2. The statement that contradicts the contents of the paragraph is...


A. Sales of the Fifty Shades of Gray novels have not surpassed Twilight and The Da
Vinci Code
B. Sales of Fifty Shades of Gray lost 50 million copies to The Da Vinci Code
C. The Da Vinci Code has the highest sales figures
D. The 100 million copy figure is the sales figure for Fifty shades of Gray worldwide
E. The Fifty Shades of Gray novel beat the sales of the James Bond series

1
SOAL-SOAL LITERASI BAHASA INGGRIS

Jawaban: B
Pembahasan:
Penjualan Novel Fifty Shades of Gray novel mempunyai selisih 100 juta copy dari
The Da Vinci Code yang berhasil mencatat angka penjualan 200 juta copy. Maka,
pilihan jawaban yang tepat adalah B.

Question No. 3 and 4 are based on this following text.

The Meteorology, Climatology and Geophysics Agency (BMKG) stated that the rainy season
will be normal until May 2013. By looking at the pattern and characteristics of rain in
Indonesia, it is estimated that tornadoes have the potential to occur until March-April 2013. in
Indonesia or around 36 percent of the total disasters during 2012," said the head of the Center
for Data, Information and Public Relations of the National Disaster Mitigation Agency
(BNPB), Sutopo Purwo Nugroho, Thursday (17/12).

According to Sutopo, the trend of tornado occurrence tends to increase every year. During
2002-2012 it increased 28-fold and there were 404 regencies/cities with a population of 115
million people living in moderate to high-prone areas for tornadoes in Indonesia. "This
condition is exacerbated by the absence of a tornado early warning system," said Sutopo.
This is due to the small range of tornadoes that fly less than 2 km, the time of occurrence is
less than 10 minutes and not all cumulonimbus clouds always occur as tornadoes. Land and
forest fires During the 2013 dry season, it has the potential to occur in 8 regular provinces,
namely North Sumatra, Riau, Jambi, South Sumatra, West Kalimantan, Central Kalimantan,
South Kalimantan and East Kalimantan." Drought has the potential to occur during August-
October in Java, Bali, NTT, and areas with a water deficit," said Sutopo.

Based on Data and Information on Indonesian Disasters (DlBl), BNPB from 1825-2012, the
number of victims who died and disappeared due to geological disasters was more than
hydrometeorological. Of the 292,330 people who died or were missing, around 74% were due
to geological disasters while 26% were hydrometeorological and others. "Society is advised
to be prepared. In Latin there is a term "si vis pacem, para bellum". This means that if you
extinguish peace, prepare for war. So even in a disaster, if you want to be safe, then prepare
for disaster. Disaster came when we were not ready. Hopefully there will be no major
disasters in the coming year," said Sutopo.

3. Statements that are not classified as facts from the text above are ...

2
SOAL-SOAL LITERASI BAHASA INGGRIS

A. The range of the tornado is less than 2 km.


B. The tornado took effect in less than 10 minutes
C. BMKG declared normal rainy season until May 2013
D. A tornado is estimated to have the potential to occur from March to April 2013.
E. 115 million people of Indonesia live in areas that are moderately prone to highly
prone to tornadoes.

Jawaban: D
Pembahasan:
Pernyataan yang tidak tergolong fakta dari bacaan diatas adalah: Puting beliung
diperkirakan berpotensi terjadi dari bulan Maret sampai dengan April 2013.
(Pernyataan ini masih berupa perkiraan yang belum tentu terjadi. Sehingga bukan
merupakan fakta).

4. The false facts below are...


A. The trend of tornado occurrence has increased 28 times in nine years
B. Tornado cases accounted for more than 30% of the total disasters in Indonesia.
C. 292,330 people died and went missing as a result of geological and
hydrometeorological disasters.
D. The number of dead and missing due to hydro-meteorological disasters is less
than geological.
E. North Sumatra, Riau, Jambi, South Sulawesi, West Kalimantan, Central
Kalimantan and East Kalimantan have the potential to experience land fires
during the dry season.

Jawaban: A
Pembahasan:
Fakta yang salah (tidak sesuai dengan bacaan) adalah: Tren kejadian puting beliung
mengalami peningkatan besar 28 kali lipat dalam tempo sembilan tahun.
(Disebutkan pada bacaan bahwa: Selama 2002-2012 (10 tahun) meningkat 28 kali
lipat dan terdapat 404 kabupaten/ kota dengan jumlah penduduk 115 juta jiwa ....)

This following text is for questions no. 5 – 9..

3
SOAL-SOAL LITERASI BAHASA INGGRIS

The Malang Regency Government is currently looking for investors to build a cement factory
in the South Malang area. It is estimated, the factory investment reached Rp. 1.4 trillion.

According to the Head of the Department of Environment, Energy and Mineral Resources of
Malang Regency Budi Iswoyo, the Malang Regency Government felt unable to mark it.
"Therefore, we really need an injection of funds from third parties who are interested in
investing here," he said, Friday (13/1).
Currently, Budi explained that his party had presented the plan to build the factory to several
cement entrepreneurs in Sulawesi through PT. Semen Gresik. But so far no one can provide
certainty.
Based on the feasibility study that has been carried out, the potential for limestone which is
the main ingredient for cement is found in five sub-districts in Malang Regency, namely in
the sub-districts of Sumbermanjing Wetan, Gedangan, Dampit and Pagak.
Secretary of the East Java Mining Companies Association (APERTAM) MH. Hudin Al-
Sonny YPE said that before thinking about establishing a cement factory, Malang Regency
Government should prioritize the development of a port in South Malang. This port has a
very important role both in terms of loading and unloading coal as factory fuel, as well as
transportation of factory production.

5. The main idea that can be concluded from the discussion above is…
A. investment needs reach Rp. 1.4 trillion.
B. Lime producing area in Malang Regency.
C. spatial planning of production centers in Malang
D. the need for investment funds Malang Regency Government
E. Malang Regency Government plans to build a cement factory in the South
Malang area.

Jawaban: C
Pembahasan:
Seluruh paragraf memaparkan rencana Pemkab Malang membangun Pabrik semen di
Malang Selatan dengan segala permasalahan.

6. Before planning the construction of a cement factory, infrastructure development that


should take precedence is...
A. port development in South Malang

4
SOAL-SOAL LITERASI BAHASA INGGRIS

B. manufacture of factory fuel coal plant


C. purchase of factory building materials
D. construction of the South Malang causeway
E. feasibility study of mapping the area of lime-producing sub-districts

Jawaban: A
Pembahasan:
Tercantum dalam paragraf ke 5

7. In the discourse above, what is meant by the investor is?….


A. PT. Semen Gresik
B. Residents of South Malang
C. Malang Regency Government
D. Third parties interested in investing
E. Association of Mining Companies (APERTAM)

Jawaban: D
Pembahasan:
Investor = pihak yang menanamkan modal di bidang usaha di daerah tertentu.

8. The main idea of the second paragraph is…..


A. development priority in Malang Regency
B. Plans to build a cement factory in South Malang
C. offering of cement factory development plan to PT. Gresik
D. South Malang as a producer of limestone as fuel for cement
E. Malang Regency Government is unable to fund the construction of a cement
factory in South Malang

Jawaban: B
Pembahasan:
Sudah jelas

9. The meaning of the word "production" in the discourse above is…


A. processing of limestone into cement
B. transportation of coal as factory fuel

5
SOAL-SOAL LITERASI BAHASA INGGRIS

C. construction of a cement factory in South Malang


D. bidding activities for the Malang Regency Government's plan to build a cement
factory to third parties
E. the inability of the Malang Regency Government to fund the construction of a
cement factory in southern Malang.

Jawaban: A
Pembahasan:
Produksi = mengolah bahan baku menjadi bahan setengah jadi atau bahan jadi.

Read the following text and answer the questions number 10 – 12.
Local media reporters covered the arrest of a Thai national who was found to be smuggling
heroin-type narcotics to Indonesia. Customs officers at Juanda Airport in Surabaya
immediately arrested the suspect who was proven to be carrying 2.671 kg of heroin.
Chanraem Suwason, was arrested at the international arrivals terminal, Thursday (23/4/2009)
night. The police suspected Chanraem Suwason was only a courier. This 27-year-old woman
is suspected of being a victim of an international narcotics syndicate. According to the
suspect's confession during police interrogation, the goods came from a native Liberian crew
member while in Thailand.

10. Parties that are not directly involved in the above case are…
A. Reporter
B. Policeman
C. Customs officer
D. Liberian ship crew
E. Chanraem Suwason

Jawaban: D
Pembahasan: Pihak yang tidak terlibat langsung dalam kasus yang disebutkan pada
bacaan tersebut adalah ABK asal Liberia. ABK asal Liberia adalah pihak yang
memberikan heroin seberat 2,671 kg kepada Chanraem Suwason dan tidak berada
tempat kejadian, yaitu bandara.

11. The reading above includes…


A. News

6
SOAL-SOAL LITERASI BAHASA INGGRIS

B. Contest
C. Clarification
D. Announcement
E. A series of events

Jawaban: A
Pembahasan:
Bacaan tersebut termasuk berita karena merupakan laporan suatu peristiwa yang
berdasarkan fakta.

12. The statement that is not in accordance with the case above is...
A. The suspect is a foreign national
B. The scene of the incident was at the airport
C. The heroin the suspect was carrying came from Liberia
D. The suspect was proven to have carried 2.671 kg of heroin
E. Police and customs officials work together to arrest the suspect

g tidak sesuai dengan kasus dalam bacaan tersebut adalah polisi dan petugas Bea Cukai bekerja sama
angka. Dalam paragraf tersebut tidak disebutkan bahwa polisi dan petugas bea cukai bekerja sama mengkap
ason.

Read the following text and answer the questions number 13 – 16.

A tapeworm is a parasite that lives in the intestines of humans and animals. Some
tapeworms attach themselves to the intestinal wall by means of suckers in their heads.
Others float freely in the intestines and absorb food through the walls of their bodies.

A tapeworm consists of numerous segments. When a new segment forms, the older ones
move to the back of the animal. Each segment contains hermaphroditic sexual organs (that
is male and female organs). The uterus of the each segment fills with eggs, which develop
into embryos. Generally, when the eggs are ready to hatch the segment breaks off and is
eliminated through the host’s excretory system. This these embryos hatch, develop in to
larvae, and grow to adults only if ingested by an in intermediate host.

7
SOAL-SOAL LITERASI BAHASA INGGRIS

One may be infected by tapeworms by eating under-cooked beef, pork, or fish. Symptoms
include irregular appetite, abdominal discomfort anemia, weakness, nervousness.

13. The passage implies that all of the following are true EXCEPT…
A. An embryo will cease to develop if not ingested by a host
B. A tapeworm will continue to live even when segments break off
C. The segment farthest back on the tail is the oldest
D. Tapeworms always float freely in the digestive system
E. A tapeworm is a parasite that lives of the human and animals.
Jawaban: D
Pembahasan: Kalimat kedua menyatakan bahwa beberapa cacing pita menempel
pada dinding usus sehingga tidak dapat mengapung dengan bebas.

14. The word “eliminated” in line 7 is closet meaning to…


A. Ingested
B. Expelled
C. Eaten
D. Grown
E. Divided
Jawaban: B
Pembahasan: Sistem ekskresi bertanggung jawab untuk membuang limbah dari
tubuh. Hanya pilihan (B) yang berkaitan dengan eliminasi.

15. A hermaphrodite is…


A. A tapeworm
B. A segment containing an embryo
C. A being that contains male and female sexual organs
D. An animal made of segments
E. A tapeworms eggs
Jawaban: C
Pembahasan: Pada paragraph kedua, kalimat ketiga, menjelaskan bahwa

8
SOAL-SOAL LITERASI BAHASA INGGRIS

hermafrodit memiliki alat kelamin laki-laki dan perempuan.

16. Which of the following statements is true?


A. A tapeworm uterus contains one egg
B. Overcooked beef is a cause of tapeworms
C. A male tapeworms must always be ingested before reproduction will occur
D. Tapeworms vary in their methods of ingesting food
E. One may be infected by tapeworms by touch the undercooked beef etc.
Jawaban: D
Pembahasan: Kita diberitahu di paragraf 1 bahwa beberapa cacing pita menempel
pada dinding usus untuk makan, sementara yang lain mengapung bebas dan
menyerap makanan melalui dinding tubuhnya.

Read the following text and answer the questions number 17 – 20.

When buying a house, you must be sure to have it checked for termites. A termite is
much like an ant its communal habits, although physically the two insects are distinct.

Like those of ants, termite colonies consist of different classes, each with its own
particular job. The most perfectly formed termites, both male and female, make up the
reproductive class. They have eyes, hand body walls, and fully developed wings. A pair of
reproductive termites founds the colony. When new reproductive termites develop, they
leave to from another colony. They use their wings only this one time and then break them
off.

The worker termites are small, blind and wingless with soft bodies. They make up the
majority of the colony and do all the work. Soldiers are eyeless and wingless but are larger
than the workers and have hard heads and strong jaws and legs. They defend the colony and
are cared for by the workers.

The male and female of the reproductive class remain inside a closed-in cell where the
female lays thousands of eggs. The workers place the eggs in cell and care of them. Even if
one colony is threated with poison, if a male and female of the reproductive class escape,
they can from a new colony.

9
SOAL-SOAL LITERASI BAHASA INGGRIS

Pest control companies can inspect a house for infestation of termites. Often, a lay
person cannot spot the evidence, so it is critical to have opinion of a professional.
Treatments vary depending upon the type of termite.

17. How the termites like ants?


A. They live in communities, and each class has a specific duty
B. Their bodies are same shape
C. The king and queen are imprisoned
D. The females reproductive capacities are the same
E. The female lays thousands of eggs and care of them
Jawaban: A
Pembahasan: Paragraf pertama mengatakan bahwa rayap dan semut memiliki
kebiasaan komunal yang serupa, tetapi mereka berbeda secara fisik. Pilihan (B) salah
karena tidak ada perbandingan tubuh semut dan rayap di bagian tersebut. Pilihan
jawaban (C) dan (D ) tidak disarankan dalam bacaan

18. The word “communal” in line 2 is closet meaning to…


A. Eating
B. Reproducing
C. Organizational
D. Social
E. Habit
Jawaban: B
Pembahasan: Kata "komunal" terkait dengan "komune" dan "komunitas", keduanya
terkait dengan bagaimana makhluk hidup berfungsi dalam kelompok "sosial".

19. Which of the following is NOT true?


A. All termite have eyes
B. Some termites cannot fly
C. Workers are smaller than soldiers
D. Termites do not fly often
E. The worker are small and blind

10
SOAL-SOAL LITERASI BAHASA INGGRIS

Jawaban: A
Pembahasan:Sudah jelas

20. The word “distinct” in line 2 is closest meaning with…


A. Similar
B. Different
C. Genetically related
D. Strong
E. Near
Jawaban: B
Pembahasan: “Distinct’ bermakna berbeda.

Read the following text and answer the questions number 21 – 24.
TEXT A
Berlin (Reuters)–No more Coca-Cola or Budweiser, no Marlboro, no American whiskey
or even American Express cards a growing number of restaurants in Germany are taking
everything American off their menus to protest the war in Iraq.
Although the protests are mainly symbolic, waiters in dozens of bars and restaurants in
Hamburg, Berlin, Munich, Bonn and other German cities are telling patrons, "Sorry, Coca-
Cola is not available any more due to the current political situation."
The boycotts appear to be part of a nascent worldwide movement. One Web site,
www.consumers-against-war.de, calls for boycotts of 27 top American firms from Microsoft
to Kodak while another, www.adbusters.org, urges the "millions of people against the war"
to "Boycott Brand America."
Consumer fury seems to be on the rise. Demonstrators in Paris smashed the windows of
a McDonald's restaurant last week, forcing police in riot gear to move in to protect staff and
customers of the American fast-food outlet. The attackers sprayed obscenities and "boycott"
on the windows.

TEXT B
This economic advantage, in turn, is used to sponsor terror and killing in Islamic
countries such as Afghanistan and Iraq. When product boycott was carried out by consumers

11
SOAL-SOAL LITERASI BAHASA INGGRIS

in the Middle East and some in the European countries, sales of these companies is reported
to be decreased by 10% and this amounted to big numbers for giant companies. Thus, it is a
rational for Muslim especially in Malaysia to take similar action. Furthermore, Of late,
many other products are available as an alternative for the boycotted products. For
example, we have Mukmin toothpaste instead of Colgate etc. and Fab or Breeze could be
replaced with Daiya, Puteri Emas etc.

There are a few questions arise in regard to the boycott such as how effective is the
approach and why not boycotting all the products altogether. In answering these questions,
Sabasun has reiterated to look at collapse of the apartheid regime in South Africa as the best
example.
To answer the question in regard to why not boycotting all the products altogether,
Sabasun has taken an approach to do what can be done when one cannot do all. Hence, a
few products that are really needed by consumers are offered on a limited floor space and
no promotion is done for the product. While products that are boycott completely will not
even get space on the shelves let alone floor space. During the early phase after the
campaign was in progress, the effect of the boycott is very obvious when Sabasun suffer a
loss of nearly RM 150,000.

21. Both passages are similar in terms of ....


A. The problem underlying in both passages
B. The writer’s point of view
C. The effect of the problem discussed
D. The area the problem taking place
E. The reason leading to the problem
Jawaban: A

22. The difference between the first and the second passage is that the latter ...
A. Asserts the economic advantage resulted from boycotting while the former
claims the cause of boycotting
B. Exposes the disserve impact of boycotting for Islamic countries while the
former shows the situation in Germany
C. Affirms the need for boycotting American products while the former depicts the
activity of boycotting in Europe
D. Repudiates the boycott toward American products while the former presents the

12
SOAL-SOAL LITERASI BAHASA INGGRIS

reason of boycotting
E. Points out the limitation of the activity while the former clarifies the need to do
the activity
Jawaban: C

23. From the second passage, it can be inferred that the topic discussed before the text B
is ...
A. The basic reason for repelling American products
B. The terror for Islamic countries
C. The motivation for not boycotting all the products altogether
D. The effectiveness in carrying out the program
E. The economic advantage of the American firms

Jawaban: E

24. The most suitable word to least change the word “nascent” in “The boycotts appear to
be part of a nascent worldwide movement.” in the first passage is ...
A. Newly born
B. Rising
C. Perilous
D. Vital
E. Fretting

Jawaban: A

Questions 25—28 are based on the following passage


Forests have the ability to regulate water systems, prevent erosion and flood, and
maintain soil fertility. The ability of forests is inseparable from the existence of millions and
even billions of trees in a forest area. Trees only store water for their own needs, and the
land saves water.
Billions of trees with their wide canopies will hold back rain. In this position, rain
water does not directly hit the soil surface, but will fall slowly through the leaves and flow
through the tree trunk. Then, the soil surface filled with tree trunks will produce quite a lot
of litter originating from organic materials in the forms of leaf and dry twigs. Dramatically,

13
SOAL-SOAL LITERASI BAHASA INGGRIS

littering is heading towards the decay process. The organic materials are collected on the soil
surface. Litter blocks the water falling from the canopy so it does not directly hit the soil.
Litter also functions as a place to live for millions of organisms (e.g. worms). This organism
punctures the land as a home and place of life. This organism’s behavior causes the soil
surface to become loose and porous. When rain drops from the canopy fall onto the litter, the
water slowly flows to the soil surface. The loose and porous top layer of soil will absorb the
water and then the water will be stored in the aquifer, the underground river.
The soil surface in the forest has high capacity to absorb rainwater. As a result, most
of the rainwater seeps into the soil; only a little becomes running water. Running water is the
water that cannot be absorbed by the soil surface. This water will go down to a lower area. If
running water exceeds the carrying capacity of the river, it can certainly cause flood. Most of
the rainwater that falls in the forest area will be absorbed by the soil and stored in the
aquifer.
Furthermore, the water stored in the aquifer will come out regularly through springs.
From these springs, water flows through rivers that are mostly found in the forest area. In
addition, there is also underground water that comes out as a spring in the resident wells.

25. The word ’dramatically’ in paragraph 2 is best replaced by ....


A. Gradually
B. Naturally
C. Amazingly
D. Surprisingly
E. Simultaneously
Jawaban: A

26. Main idea of the passage is….


A. Water guarantees millions of organisms to survive in forest areas.
B. Land stores and releases water through various processes.
C. Litter gradually flows water to the soil surface.
D. Trees store water to fulfill human daily needs.
E. Forests have various benefits for human life.
Jawaban: B

14
SOAL-SOAL LITERASI BAHASA INGGRIS

27. With the sentence ‘This organism's behavior causes the soil surface to become loose
and porous’ in paragraph 3, the writer intends to ....
A. Describe the organism and its behavior against loose soil
B. Confirm the behavior of organisms in loose soil
C. Compare the loose soil with the porous soil
D. Uncover natural processes at soil erosion
E. Explain the causes of soil erosion

Jawaban: E

28. What most likely motivates the writer in writing the passage?
A. There is an unresolved flood problem.
B. People’s awareness towards reforestation is low.
C. Not many people understand the benefits of water for forests.
D. There is the fallacious concept that trees store large amount of water.
E. The information about the importance of trees in storing water is not available.
Jawaban: D

The text is for the following question.


Remote sensing is the acquisition of information about an object or phenomenon,
without making physical contact with the object. In modern usage, the term generally refers
to the use of aerial sensor technology to detect and classify object on Earth (both on the
surface and in the atmosphere and oceans) by means of propagated signal again (e.g
electromagnetic radiation emitted from aircraft satellites).

There are two main types or remote sensing: passive remote sensing and active
remote sensing. Passive sensors detect natural radiation that’s it emitted or reflected by the
object or surrounding area being observed. Reflected sunlight is the most common source of
radiation measured by passive censors. Examples of passive remote sensors include film
photography, infra-red, charge-coupled device and radiometers. Active collection, on the
other hand, emits energy in order to scan objects and areas whereupon a sensor then detect
and measures the radiation that is reflected or back scattered from the target. Radar and Lidar
are examples of active remote sensing where the time delay between emission and returns is
measured, stabilizing and location, height, speed and direction of an object.

15
SOAL-SOAL LITERASI BAHASA INGGRIS

29. From the text we known that remote sensing ….


A. Does not difficult to do
B. Does not make use of sensors
C. Does not make use propagated signals
D. Does not makes physical contact with the object
E. Does not information about an object or phenomenon

Jawaban: D

The text is for the following question


Mr. Hartoyo who has become a widower for almost 5 years has three children : two
sons and one daughter. All of them are already adults. The two sons are now studying at a
university in Yogyakarta. While his only daughter lives with him in a small town in Central
Java is in the third year of SMU. Martuti,  the only daughter, plans to continue her study in
Yogya, too, after graduate from SMU. She never stopped persuading her father to let her go
to Yogya. However on the contrary her father totally disagrees to what she wants since she is
the only daughter and if she leaves nobody will company him in his old age. As an obedient
one, she is afraid to say “No” to what is said by her father although in her heart she really
wants to rebel. In her opinion, her father is absolutely discriminative.

30. The communicative purpose of the text ….


A. To share an amusing story
B. To inform factual information
C. To criticize Mr. Hartoyo character
D. To describe Mr. Hartoyo’s character
E. To entertain and deal with actual information

Jawaban: B

The text is for the following question.


Slang is definitely a way to distinguish you as part of a group or separate from another
group. Teens use slang to show their independence from their parent, and give them and their
friends a language they can call their own. Slang sometime allows them to communicate
whithout outsider understanding, or to show off that they belong to a particular group. Teens
like to use slang word for several reasons.

16
SOAL-SOAL LITERASI BAHASA INGGRIS

Firstly, they use slang to show their individuality. Daily teen conversations can be
incomprehensible to many parents and adults. Influenced by MTV, hip-hop culture and the
popularity of e-mail, teenagers shorten words, stretch definitions and pronounce term with a
different style. The way the talk shows their confidence.

Secondly, they use slang because it is easier to say. They might talk the way they do
with word like "coz" for "because" and "holla" for "hello" and other expressions because the
words are cool and easier to use, not because the are lazy.

Finally, they use slang for speed and usefulness, and little humor of the day. Slang
flows quicker than standard language. Slang can color their vocabulary and sentences, so that
they don't seem boring.

Thus, from the reason above, it can be concluded that teens use slang words because
using their own terms is the easiest way to make the world of their own.

31. What is the purpose of the text?


A. To describe what slang is
B. To persuade the readers to use slang
C. To explains the readers about what slang is
D. To inform the readers about why teens using slang
E. To present two different points of view about slang

Jawaban: D

Choose the best answer!

To the Editor of the Herald:

The U.S. House of Representatives has recently proposed a law (H.R. no. 396) that will
ban the sale of cigarettes from vending machines. This is a landmark piece of legislation
that everyone in our community should support right now.
Many people don't realize it, but vending machines are one of the main places that children
are able to purchase cigarettes. In fact, it is estimated that 10% of all cigarettes purchased

17
SOAL-SOAL LITERASI BAHASA INGGRIS

by minors take place at these machines. If this new legislation goes into effect, it will not
only make it more difficult for youth to break the law by buying cigarettes, it will lower the
chances of young people smoking in the first place.
There are many reasons (besides the legal ones) why we should try to curb smoking by our
children. 1) Research shows that most people who smoke started when they were underage.
2) Many people fear that smoking cigarettes serves as a "gateway" to harder drugs. 3)
Smoking is a very expensive addiction (particularly for a teen who makes minimum wage)
And, finally, 4) smoking can cause many life-shortening or fatal health problems (such as
lung cancer and emphysema). Our young people would not smoke before they are really
able to understand or accept the long- term consequences.
The vending-machine bill has been proposed, but now it needs to be passed. Your voice
will count here. We encourage you to write or call the representative for your district (for
those of us in District 8, that's Congresswoman Fisher) and let her know that you support
her as she tries to get this legislation passed. The more support she gets, the more likely it
is that this bill will become law. Contact Tobacco Free Youth for further information about
this important issue.

Sincerely.
Jonathan Friedman, Director
Tobacco Free Youth
123 Forest Road

32. What does Jonathan want with the letter…


A. To argue that the recently proposed law is irrelevant with present condition.
B. To make public aware of the danger of vending machines selling cigarette.
C. To comment on a law that will ban the sale of cigarette from vending machines.
D. To list arguments to oppose the ban of selling of cigarette from vending
machines.
E. To gain support from public about a law that will ban the sale of cigarette from
vending machines.
Jawaban: E

18
SOAL-SOAL LITERASI BAHASA INGGRIS

33. Which statement is true based on the text ….


A. Vending machines always sell cigarette
B. In order that the bill become law, it need to be supported by public
C. Jonathan Friedman is the director of The U.S. House of Representative
D. Everybody agrees with the ban of selling of cigarette from vending machines
E. The bill concerning the ban of selling of cigarette from vending machines has
been applied
Jawaban: B

The text is for the following question.

Jakarta post:

The research, published in the proceedings of the Royal Society B, has investigated
the bacterium in eleven individuals ranging from as far back as the Bronze Age up to the
twentieth century. Samples have been taken from individuals in Europe and in both pre and
post-colonial America. The oldest case study was that of an individual from 1200 BC, found
in the Burial cave in Montainsell (Licidia, Catalonia).

The study found that ancient genetic material from principle Bacterium, streptococcus
mutant, causes dental carries. The mutant has increased its genetic material change overtime.
This is possibly due to the coincidence of dietary changes that are linked with the expansion
of humanity.

Similar study conducted by researchers from the University Autonma de Barcelona


(UAB) and the laboratoria Nasional de Genemica para la Bie diversided, Mexico, put the
genetic material in order and found that genetic diversity has been produced, especially in the
fragment of a gene that coldifies a virulence factor known as dextranase.

34. What is the first paragraph about ….


A. The research genetic diversity.
B. The investigation of Bacterium.
C. The investigation of dental carries.
D. The samples of individuals from pre- colonial Europe.
E. The investigation of the bacterium in eleven individuals.

19
SOAL-SOAL LITERASI BAHASA INGGRIS

Jawaban: E

Questions on the following passage

The impacts of electronic commerce in a developing country can be helpful rather


than detrimental. Electronic commerce has the potential to tie developing countries into the
rest of the world so they are no longer considered outsiders. For example, electronic
commerce can enable more people to access products and services that once were not
available. Another benefit is that electronic commerce stores are available 24 hours a day, 7
days a week. As the infrastructure for electronic commerce keeps growing, services that were
not offered in the past become available. Many of these benefits have not been proven yet,
but the technology is now available, and developing countries are looking forward to these
benefits. The high cost of technology may still be detrimental in many developing countries,
however, the constant innovation of software and hardware will hopefully reduce these costs.

Consumers in developing countries can benefit from electronic commerce because


they can buy product that could only be found in major cosmopolitan cities. Electronic
commerce is closing the gap between the countries that have wide availability of products
and those with limited availability. The basic purpose of electronic commerce is to provide
goods and services to consumers who do not live close to the physical location of the product
or service and would otherwise have a hard time acquiring these products and services.

Society and consumers alike have only begun to enjoy the benefits of electronic
commerce. Since new developments are made on a continuous basis, it will eventually affect
every individual. Some of the benefits enjoyed by society and consumers, for example, are
ease of transaction, comparability of products, quick delivery and the ability to make any type
of transaction at any given time of day.

Electronic commerce facilitates delivery of public and social services, such as


healthcare, education, and distribution of government social services at a reduced cost,
improving the quality of care and living in these communities. For example, healthcare
services can reach patients in rural areas.

35. The topic of the text is….


A. Government policy on electronic commerce.
B. Types of business suitable for electronic commerce.

20
SOAL-SOAL LITERASI BAHASA INGGRIS

C. The benefits of electronic commerce for developing countries.


D. The gap between countries applying and not applying electronic commerce.
E. Minimum requirements of software and hardware for electronic commerce.
Jawaban: C

Questions on the following passage

European society in the 18th century was a broad pyramid, with the few of the nobility at the
top and the masses of the peasantry at the bottom. In western Europe there was an increasing
split between the wealthy nobility, who spent much of their time practicing exquisite etiquette
at court, and those lower nobles who stayed in the countryside, hunting and running their
estates with little concern for either social niceties or abstract ideas. Only a few at the social
peak lived the life protrayed in 20th century historical novels-amid bright chandelers,
powdered wigs, and beautiful women-but it was in this mileu that the century earned its
reputation Forlicentiousness and decadence. Every monorach had his mistresses, and the
ways f Versailles were mimicked across Europe, most absurdly in the courts of the tiny
German pricipalities.

36. The paragraph which precedes this one most likely discusesses….
A. Social problems
B. European lower nobles
C. Family disputes in Europe
D. Wealthy people in Germany
E. Social class in European society
Jawaban: E

Read the text below to answer the question number 37 – 41.

In the following article, the author speculates about a connection between the
law-fat, high- carbohydrote diet recommended by the medical establishment in the last
twenty years and the increasing rate of obesity among Americans.

American dietitians and members of the medical community have ridiculed low-
carbohydrate diets as quackery for the past thirty years, while extolling a diet that cuts down
on fat, limits meat consumption, and relies on carbohydrates as its staple. Many Americans

21
SOAL-SOAL LITERASI BAHASA INGGRIS

are familiar with the food pyramid promoted by the U.S. government, with its foundation of
carbohydrates such as breads, rice, and pasta, and its apex allotted to fats, oils, and sweets.

Adhering to the government's anti-fat, pro- carbohydrate gospel, food manufacturers


have pumped out fat-free grain products that lure consumers with the promise of leaner
days. Then, why are Americans getting so fat? Could the dietary recommendations of the
last twenty years be wrong? And what's more, could the proponents of diets that push
protein and fat be right?

Fact: Obesity rates have soared throughout the country since the 1980s. The United
States Centers of Disease Control reports that the number of obese adults has doubled in the
last twenty years. The num-ber of obese children and teenagers has almost tripled,
increasing 120% among African- American and Latino children and 50% among white
children. The risk for Type 2 diabetes, which is associated with obesity, has increased
dramatically as well.

Disturbingly, the disease now affects 25% to 30% of children, compared with 3% to 5% two
decades ago.

What is behind this trend? Supersized portions, cheap fast food, and soft drinks combined
with a sedentary lifestyle of TV watching or Internet surfing have most likely contributed to
the rapid rise of obesity. Yet, there might be more to it: is it a coincidence that obesity rates
increased in the last twenty years-the same time period in which the low-fat dietary doctrine
has reigned? Before the 1980s, the conventional wisdom was that fat and protein created a
feeling of satiation, so that overeating would be less likely. Carbohydrates, on the other hand,
were regarded as a recipe for stoutness. This perception began to change after World War II
when coronary heart disease reached near epidemic proportions among middle-aged men. A
theory that dietary fat might increase cholesterol levels and, in turn, increase the risk of heart
disease emerged in the 1950s and gained increasing acceptance by the late 1970s. In 1979,
the focus of the food guidelines promoted by the United States Depart-ment of Agriculture
(USDA) began to shift away from getting enough nutrients to avoiding excess fat, saturated
fat, cholesterol, and sodium-the components believed to be linked to heart disease. The anti-
fat credo was born.

To date, the studies that have tried to link dietary fat to increased risk of coronary
heart disease have remained ambiguous. Studies have shown that cholesterol-lowering drugs

22
SOAL-SOAL LITERASI BAHASA INGGRIS

help reduce the risk of heart disease, but whether a diet low in cholesterol can do the
same is still questionable. While nutrition experts are debating whether a low-fat,
carbohydrate-based diet is the healthiest diet for Americans, nearly all agree that the anti-fat
message of the last twenty years has been over-simplified. For example, some fats and oils
like those found in olive oil and nuts are beneficial to the heart and may deserve a larger
propor-tion in the American diet than their place at the tip of the food pvra-rnid indicates.
Likewise, some carbohydrates that form the basis of the food pyramid, like the "refined"
carbohydrates contained in white bread, pasta, and white rice, are metabolized in the body
much the same way sweets are.

According to one Harvard Medical School researcher, a breakfast of a bagel with low-
fat cream cheese is "meta-bolically indistinguishable from a bowl of sugar."

So what about those high-fat, protein diets that restrict carbohy-drates like the popular
Atkins' diet and others? A small group of nutrition experts within the medical establishment
find it hard to ignore the anecdotal evidence that many lose weight successfully on these
diets. They are arguing that those diets should not be dismissed out of hand, but researched
and tested more closely. Still others fear that Ameri-cans, hungry to find a weight-loss
regimen, may embrace a diet that has no long-term data about whether it works or is safe.
What is clear is that Americans are awaiting answers and in the meantime, we need to eat
something.

37. The passage is primarily concerned with...


A. questioning the dietary advice of the past two decades.
B. contrasting theories of good nutrition.
C. displaying the variety of ways one can interpret scientific evidence.
D. debunking the value of diets that restrict carbohydrates.
E. isolating the cause of the rising rate of obesity.

Jawaban: A
Pembahasan: The article raises the question, Could the dietary recommendations of
the last twenty years be wrong? (lines 11-12).

38. The passage suggests that the obesity trend in the United States is...
A. partly a result of inactive lifestyles.

23
SOAL-SOAL LITERASI BAHASA INGGRIS

B. the predictable outcome of cutting down on saturated fat.


C. a cyclical event that happens every twenty years.
D. unrelated to a rise in diabetes cases.
E. the unfortunate byproduct of the effort to reduce heart disease.
Jawaban: A
Pembahasan: The author names a sedentary lifestyle of TV watching and Internet
surfing (Paragraph 4:lines 2-3) as a contributing factor to the rise in obesity rates

39. In the fourth paragraph, the author implies that the government's 1979 food guidelines...
A. relied more on folk wisdom than on scientific study.
B. was based on the theoretical premise that eating less dietary fat reduces heart
disease.
C. was negligent in not responding to the increasing incidence of heart disease.
D. no longer bothered to mention nutrient objectives.
E. was successful in reducing heart disease rates
Jawaban: B
Pembahasan: The passage suggests that the 1979 dietary guidelines responded to a
theory that dietary fat (line 13) increases heart disease.

40. The author cites the example of a breakfast of a bagel with low-fat cream cheese in
order to...
A. show that getting a nutritional breakfast can be fast and convenient.
B. demonstrate that carbohydrates are the ideal nutrient.
C. overturn the notion that a carbohydrate-based breakfast is necessarily healthy.
D. persuade readers that they should eat eggs and sausage for breakfast.
E. argue that Americans should greatly restrict their carbohydrate intake.
Jawaban: C
Pembahasan: This example supports the claim that the body uses refined carbo-
hydrates in much the same way (Paragraph 5:lines 10-11) that it does sweets.

41. The tone of the last sentence of the passage is one of...
A. optimism.
B. resolve.

24
SOAL-SOAL LITERASI BAHASA INGGRIS

C. indulgece.
D. irony.
E. revulsion.
Jawaban: D

Pembahasan: The last sentence is ironic-it expresses an incongruity between


conflicting dietary advice that targets different types of food as unhealthy, and the
reality that humans need to eat.

Read the text below to answer the question number 42 - 43

The following passage is from Frank McCourt's 1996 memoir Angela's Ashes. The author
describes what it was like to go to school as a young boy.

We go to school through lanes and back streets so that we won't meet the respectable
boys who go to the Christian Brothers' School or the rich ones who go to the Jesuit school,
Crescent College. The Christ-ian Brothers' boys wear tweed jackets, warm woolen sweaters,
shirts, ties, and shiny new boots. We know they're the ones who will get jobs in the civil
service and help the people who run the world. The Cres-cent College boys wear blazers and
school scarves tossed around their necks and over their shoulders to show they're cock o' the
walk. They have long hair which falls across their foreheads and over their eyes so that they
can toss their quaffs like Englishmen. We know they're the ones who will go to university,
take over the family business, run the government, run the world. We'll be the messenger
boys on bicycles who deliver their groceries or we'll go to England to work on the building
sites. Our sisters will mind their children and scrub their floors unless they go off to
England, too. We know that. We're ashamed of the way we look and if boys from the rich
schools pass remarks we'll get into a fight and wind up with bloody noses or torn clothes.
Our masters will have no patience with us and our fights because their sons go to the rich
schools and, Ye have no right to raise your hands to a better class of people so ye don't.

42. The passage suggests that the author goes to school...


A. in shabby clothing.
B. in a taxi cab.
C. in warm sweaters and shorts.
D. on a bicycle.

25
SOAL-SOAL LITERASI BAHASA INGGRIS

E. to become a civil servant.


Jawaban: A
Pembahasan: The author and his classmates go to school through lanes and back
streets {line 1) to avoid the students who go to school dressed in warm and
respectable clothing. He also states in lines 15-16 that they are ashamed of the way
we look, implying that they are poorly dressed.

43. The word pass as used in line 14 means to...


A. move ahead of.
B. go by without stopping.
C. be approved or adopted.
D. utter.
E. come to an end.
Jawaban: D
Pembahasan: The boys would get into fights if the rich boys were to utter
derogatory words or pass remarks.

Read the text below to answer the question number 44 – 48.

The Covid-19 vaccine has changed the course of the pandemic. In total, 158 doses
have been given for every 100 people around the world. In countries that reached the highest
levels of vaccination, death rates plummeted.
Amid this progress, the world was caught off guard when the omicron variant rose to
prominence in late 2021. Omicron proved somewhat less severe than previous variants, but
it spread much faster.
Millions of vaccinated people have gotten sick from the omicron strain, but the shots
served their most important function-preventing severe illness. During the omicron wave,
vaccination with a booster reduced the chance of hospitalization and death by more than
90%. Sudden spikes of cases from the fast- moving variant have pressured health systems,
but the outlook for the average Covid patient has improved significantly.
In early 2021, Israel was first to show that vaccines were bending the curve of Covid
infections. The country led the world in early vaccinations, and cases declined rapidly. A
similar pattern of vaccination and recovery was repeated across dozens of countries.

26
SOAL-SOAL LITERASI BAHASA INGGRIS

New, more transmissible variants caused renewed outbreaks, but in these subsequent
waves, unvaccinated patients faced far higher risk of hospitalization and death. This
disparity in outcomes led health officials to dub the current phase of Covid a "pandemic of
the unvaccinated."
Since the start of the global inoculation campaign, countries have experienced unequal
access to vaccines and varying degrees of success in getting shots into people's arms.
This chart shows how Covid cases and deaths compare with prior waves in each
country. When omicron took off in late 2021, cases soared but were less likely to be fatal.

44. How does paragraph 1 relate to paragraph 2 according to the passage...


A. high vaccination rates have caught the world off guard in tackling the fast spread
of omicron
B. many people died because of the vaccination and also the rapid spread of omicron
C. in total, 158 doses have been administered for every 100 people in worldwide to
tackle the fast-spreading spread of omicron.
D. all countries get vaccinated fast because of the fast spread of omicron too.
E. low vaccination rates make the spread of omicron spread rapidly making many
people die
Jawaban: A
Pembahasan: paragraf pertama membahas tentang suksesnya proses vaksinasi yang
telah dilakukan di banyak negara. Sedangkan di paragraf kedua membahas tentang
virus omicron yang terjadi dan itu tidak diantisipasi oleh banyak negara, walaupun
omicron tidak terlalu parah dibandingkan dengan virus sebelumnya, namun

27
SOAL-SOAL LITERASI BAHASA INGGRIS

penyebarannya lebih cepat.


Kesirnpulannva, vaksinasi yang tinggi membuat beberapa negara lengah dengan
kemuncululan virus omicron, sehingga jawabannya A.

45. Which of the following is the best paraphrase of the sentence "Sudden spikes of cases
from the fast- moving variant have pressured health systems, but the outlook for the
average Covid patient has improved significantly''?
A. the increased immunity due to the previous virus provides high prospects
B. the case of the virus variant has given a system change, but the community can't
survive
C. the surge in cases of the virus makes the prospects of covid patients increase but
it is not controlled
D. the vaccination process that has been carried out by many patients has changed
the social system
E. the previous high cases of the virus had an effect on health status, but the majority
of patients exposed to the virus survived

Jawaban: E
Pembahasan: Inti dari kalimat di atas adalah lonjakan kasus yang tiba-tiba dari
varian yang bergerak cepat telah menekan sistem kesehatan, tetapi prospek rata-rata
pasien Covid telah meningkat secara signifikan. Jika di paraphrase mengandung
makna bahwa kasus virus yang
tinggi sebelumnya berdampak pada status kesehatan, tetapi sebagian besar pasien
yang terpapar virus selamat. Hal itu sesuai dengan jawaban E.

46. The word "outlook" in sentence 5 paragraph 3 refers to...


A. Covid·19
B. Healthy
C. Antibody
D. Patient
E. Vaccination

Jawaban: C

28
SOAL-SOAL LITERASI BAHASA INGGRIS

Pembahasan: outlook yang dimaksud dalam paragraf tersebut merupakan ketahan


tubuh atau antibody patient yang telah terpapar covid-19 sebelumnya

47. Based on the note that follow the graph above, we can understand that...
A. the number of omicron cases is directly proportional to the number of deaths in
2022
B. Covid-19 cases until early 2021 have increased the number of deaths with the
number of vaccinations not being maximized
C. after maximum vaccination, the death rate decreased even though omicron
cases soared until 2019
D. after maximal vaccination, the mortality rate decreased even though omicron
cases soared until 2022
E. the peak of new Omicron cases occurred in January 2022 which caused the
number of deaths to be unpredictable
Jawaban: D
Pembahasan: sesuai dengan graph dan paragraph di atas, dapat dipahami bahwan
setelah vaksinasi maksimal, angka kematian menurun meski kasus omicron
melonjak hingga 2022. Walaupun angka kasus meningkat namun jumlah kematian
tidak terlalu tinggi. Selain itu, pembahasan paragraph di atas juga membahas
tentang pemaksimalan dan berhasilnya proses vaksinasi, jadi jawaban yang tepat
adalah D.

48. Which of the following sentence about In early 2021 in Israel is NOT CORRECT
according to the passage?
A. the country carried out the first vaccination and reduced the number of cases of
the spread of Covid-19
B. Israel's vaccination process is followed by many countries
C. the first vaccine is made in 2021 in Israel
D. the process of the vaccination pattern was successfully carried out by Israel and
provided a recovery for the number of cases of the spread of Covid
E. world leading country in early vaccination
Jawaban: C
Pembahasan: Israel tidak pernah membuat vaksin pertama di tahun 2021 sesuai

29
SOAL-SOAL LITERASI BAHASA INGGRIS

dengan isi paragraph.. Pilihan ganda A·E merupakan isi paragraph 4 kecuali pilihan
ganda C.

Read the text below to answer the question!


Shop owners are losing a lot of money because of the shoplifting that happens so
frequently nowadays. The question now is wheather first offenders should be let off lightly.

On the one hand, it is not fair to punish people the first time they make a mistake. The
police should talk sternly to them and give them a warning.

On the other hand, every day, shops lose thousands af dollar’s worth of valuable items.
This effects us all because shop owners wll increase prices, to counter the loss.
Consenquently, we have to pay extra for the loss.

In my opinion, first offenders should be warned to make them feel guilty and to shop their
unlawful deeds. Punishment should also depend on some factors such as their age, the
reasons why they shoplifted and the family background.

49. What is the purpose of the text?


A. To argue that shoplifters should be punished harshly
B. To argue that first time shoplifters be given leniency
C. To persuade readers to punish shoplifters lightly
D. To discuss the penalty for first time shoplifters
E. To persuade readers to forgive shoplifters
Jawaban: D
Pembahasan:
Untuk mampu mengetahui tujuan dari teks, kita harus mampu mengidentifikasi ide-
ide utama pada teks tersebut. Dalam teks tersebut terdapat dua pandangan tentang
hukuman untuk first time shoplifters ( pengutil yang baru sekali mencuri). Maka
tujuan dari teks adalah untuk mendiskusikan hukuman bagi para first time shoplifter.

Read the text below to answer the question


Attention, all residents! Due to the long drought, the Central Water Authority advises you to
use less water. There is very little water in the reservoir, and rain is not expected until the end
of this year. For more information, please visit the city government website. There you will

30
SOAL-SOAL LITERASI BAHASA INGGRIS

find a list of ways to reduce water consumption, including watering indoor plants with
dishwater and taking showers instead of baths.

50. What is the announcement about….


A. warning of decreasing water in a reservoir
B. Advise to increase water in a reservoir
C. Advise for residents to use less water
D. Warning of water consumption
E. Warning of long drought

Jawaban: C
Pembahasan:
Disebutkan dalam pengumuman bahwa dikarenakan musim kemarau panjang maka
dianjurkan untuk menghemat air (“the central water authority advises you to use less
water”).
Maka dapat disimpulkan pengumuman tersebut mengenai anjuran untuk menghemat
air.

Read the text below to answer the question!

Last week, we made vegetable stew at school. We brought some meats and vegetables. We
washed the vegetables. Some of us cut the meat into pieces. First, we put the meat into the
pot with a small amount of cold water. We cooked the meat at simmering temperature. After
a while, we put the vegetables and finally the noodles into the pot. We waited for about
fifteen minutes for it to cook.

51. Which of the following is first cooked….


A. Vegetables
B. Noodles
C. Cheese
D. Meat
E. Fruit

Jawaban: D
Pembahasan:

31
SOAL-SOAL LITERASI BAHASA INGGRIS

Bahan yang pertama dimasak adalah daging karena dalam monolog tersebut
dikatakan”First, we put the meat into the pot”.

Read the text below to answer the question!


My name is Zandile Mzazi. I’m from South Africa. When I was a little girl, I loved
to sing. I tried to sing along with stars like Whitney Houston. I got professional start singing
religious music called oratorio, which is like opera, but has religious themes, for example
Handel’s “Messiah”. I first heard that kind of music from my aunt. I could sing the “Messiah”
from the age of nine. I listened to opera recordings and I imitated the sound that I was
hearing. As a teenager, I heard a South African opera singer on television. I started to sing
along that is how I discovered opera. One thing I like about opera is its sadness and drama.
Today, I’m a professional opera singer. Last July, I went to Europe for a voice
competition and came first out of 80 singer from around the world. I have sung in front of
Nelson Mandela and the president of China. I’m planning to go to Europe to continue my
career, but South Africa is still home to me.

52. What is the main idea of paragraph 1?


A. Handel’s “Messiah”
B. Zandile Mzazi’s singing career
C. Religious music called oratorio
D. The start of Mzazi’s passion for singing
E. A south African opera singer on television
Jawaban: D
Pembahasan:
Untuk menemukan gagasan pokok dari suatu paragraph, kita harus mampu
menemukan topic sentence dari paragraph tersebut. Topic sentence terdapat pada
bagian awal paragraph atau bagian akhir paragraph. Pada teks tersebut, topic
sentence pada paragraph pertama terletak pada bagian awal paragraph yakni “I got
my professional start singing religious music called Ontario…”
Read the text below to answer the question!

SCHOOL EXCURSION TO GOLDEN SANDS BEACH


Saturday, 20 Desember (The beach is about 20 kilometers west)

32
SOAL-SOAL LITERASI BAHASA INGGRIS

Bus departure time : 7.30 AM


Arrival time : 9.00 AM
Programmes :
Morning : swimming, games, volleyball, lunch in a seaview
restaurant
Afternoon: walking along the beach to the lagoon, watching the
boat festival.

53. What can we infer from the text?


A. Golden Sands Beach is located to the west of the city
B. It takes four hours to go to the beach
C. The bus will stop at the lagoon
D. The excursion is on a weekday
E. The are boarding a boat
Jawaban: A
Pembahasan :
Untuk menjawab pertanyaan tersebut, kita harus mencocokkan pilihan – pilihan
jawaban dengan informasi yang ada pada teks tersebut. Jawaban yang sesuai dengan
informasi yang ada pada teks adalah Golden Sands Beach is located to the west of
the city karena sesuai berdasarkan informasi dari teks, “The beach is about 20
kilometers west”.

Read the text below to answer the question!


JOIN OUR SCHOOL ENGLISH DEBATE
Further to our school vision : “Excellent in Academic Achievement at National and
International Levels”, we will conduct an English debate competition for 10th and 11th
graders.
 Each class should send a team (3 students). The students should be recommended by the

33
SOAL-SOAL LITERASI BAHASA INGGRIS

respective English teacher.


 A technical meeting will be held on 3 Oktober 2019 at AV room at 3.30 PM to 4.30
PM.
 The competition will be held on 8 October 2019 at 8.00 AM.
 Based on the presentation, adjudicators will choose the three best speakers.
 Selected best speakers will be awarded prizes and certificates.

54. Who decides which students will represent their classes?


A. The students
B. The classmates
C. The headmaster
D. The adjudicators
E. Their English teachers
Jawaban: E
Pembahasan :
Yang berhak menentukan siswa yang mewakili dalam lomba debat adalah guru
bahasa inggris. Karena didalam teks dituliskan The students should be recommended
by the respective English teacher

Read the text bellow to answer the questions!

When people start stargazing with a telescope, they are typically first astonished by the view.
Saturn’s golden rings appear in fine detail, star cluster glitter like jewelry on black velvet,
galaxies glow with gentle starlight older than the human species. The view brings home the
realization that we and our world are an internal part of a gigantic system. That’s the initial
reaction. The next is a desire for a bigger telescope.

The first person to point a telescope at the night sky was Galileo, roughly 400 years ago.
With his telescope, Galileo was able to chart four bright satellites as they moved around
Jupiter, like planets in a miniature solar system- something that contemporary critics had
dismissed as physically impossible.

Galileo’s telescope also revealed thousands of previously invisible stars. When he tried to
map all of them in just on constellation, Orion, he gave up, confessing that he was over”

34
SOAL-SOAL LITERASI BAHASA INGGRIS

whelmed by the vast quantity of stars.” Evidently the Earth was a small part of a big universe,
not a big part of a small one. Some six decades later, Isaac Newton pioneered a new kind of
telescope, the reflecting telescope. This made it possible to obtain a much clearer view of the
universe. Mirrors required that only one surface be used to reflect star light to a focal point.
Since the mirror was supported from behind, it could be quite larger without staging under its
own weight, as large as lenses tended to do.

It was with such a reflecting telescope that William Herschel discovered the planet Uranus in
1781. Six decades later, Lord Rosse built a massive reflecting telescope called Leviathan on
his estate in Ireland. This allowed Rosse to become the first person to observe spiral-armed
galaxies beyond the Milky Way.

55. The main point of this text is to …


A. explain an important new development of astronomy
B. contrast ground-based telescope and spacebased telescope
C. trace the development of telescopes
D. discuss Galileo’s and Newton’s contribution to the field for astronomy
E. show the different between Galileo’s and Newton’s telescope
Jawaban: D
Pembahasan:
Setiap paragraf pada teks ini menceritakan mendiskusikan kontribusi Galileo dan
Newton di bidang Astronomi.

56. According to the text above, the use of phrase gave up in the text indicates that Galileo
was …
A. upset
B. discourage
C. successful
D. curious
E. desire
Jawaban: B
Pembahasan:
gave up = menyerah, discouraged = berkecil hati. Hanya kata discourage yang

35
SOAL-SOAL LITERASI BAHASA INGGRIS

mengandung arti yang mirip dengan gave up.

Read the dialogue!

Farhan : So, how long has it been since the first publishing of your first novel?
Yola : It’s been 3 weeks since then.
Farhan : So, how many copies have been sold this far?
Yola : Thank God. Many people seem to like it very much. Since the first
Publishing weeks ago, there have been over 500 copies sold.
Farhan : Splendid! I’d like to congratulate you on your success at your first time
publishing your novels.
Yola : ..........................
Farhan : Keep the good work, Yola. I know you could be a great author someday.

57. What Yola said after get a compliment from Farhan?


A. I feel this is make me very shocked
B. Thank you very much, Mr. Farhan. It’s very kind of you to say so.
C. Thank you very much, Mr.Farhan. it’s very bad of you to say so
D. Thank you very much, Mr. Farhan. My dress is created by Muflika.
E. Thank you very much, Mr.Farhan. this green shoes make me taller than before.
Jawaban: B
Pembahasan:
Dari pernyataan berikut “Splendid! I’d like to congratulate you on your success at
your first time publishing your novel” menunjukkan bahwa Farhan memberikan
penghargaan terhadap pencapaian penerbitan novel pertama Yola.

Read the Text bellow to answer question number 58 – 60.

Forget lions, tigers, and bears. When it comes to the art of war, army ants are among
the most frightening creatures on earth. With powerful mouth parts, these fighters can
skilfully cut creatures much larger than themselves into pieces. Acting together in great
numbers, army at colonies succed at making tens of thousands of such kills each day. Their

36
SOAL-SOAL LITERASI BAHASA INGGRIS

capabilities do have limits, though. Contrary to popular belief, they almost never take down
large animals or people.
One of the best places to observe army ants is Barro Colorado, an island in a lake
created by Panama Canal. The island is home to as many as 50 colonies of Eciton burchellii,
the most studied army ant in the world. It is one of 150 types of army ants in the New World;
more 170 other types live in Asia, Africa, and Australia.
The colonies of this army ant are huge, ranging from 300.000 to 700.000 ants. They
never stay in one place long, moving from nest site to nest site. Linking legs together, they
use their own bodies to form enormous nests called bivouacs, which they hang beneath a
fallen tree. There they stay for about 20 days as the queen lays as many as 300.000 eggs.
When the ants go hunting as many as 200.000 of them leave the nest in a group that
broadens into a fan as wide as 14 meters. This swarm raid takes a slightly different course
each day, allowing the hunter to cover fresh ground each time.
Protecting the ants wherever they go are soldiers, recognizable by their oversized
jaws. If their frightening looks do not scare enemies away, soldiers also have a powerful bite
and the attack is often suicidal. Because their jaws are shaped like fishhooks, the soldiers
cannot pull them out again. Amazonian tribes have used soldier ants to close wounds,
breaking off the bodies and leaving the head in place.
Eciton burchellii are blind and cannot see what is a head of them, but they move
together in such great numbers that they can easily kill the non army ants, insect and other
small creatures that constitute their prey. When the groups happen upon a break in the path,
ants immediately link legs together and form living bridges so that the groups can move
forwards without any delay.
In Japanese the word ant is written by linking two characters: one meaning “insect”,
the other meaning “loyalty”. Indeed, individual ants are completely loyal to their fellow ants.
They display many examples of selfless cooperation that, while certainly extreme, cannot fail
to win human admiration.

58. The author’s purpose of writing the text is to ….


A. Inform the readers about how army ants set up their life
B. Demonstrate the strength of army ants compared to lions, tigers, and bears
C. Analyze how soldier ants protect their colonies
D. Make the readers aware of the use of army ants
E. Refute a common misconception about army ants

37
SOAL-SOAL LITERASI BAHASA INGGRIS

Jawaban: E
Pembahasan:
Tujuan utama penulis dalam menuliskan teks ini adalah untuk menyangkal (refute)
anggapan bahwa semut adalah binatang yang lemah, karena sebenarnya mereka
bahkan lebih kuat daripada binatang buas seperti harimau dan beruang

59. How is the information in the last paragraph organized?


A. A statement is followed by examples and explanation.
B. A statement is followed by research findings.
C. A statement is followed by explanation organized from general to specific.
D. A statement is followed by supporting ideas organized in cause and effect.
E. A statement is followed by definition and explanation.
Jawaban: D
Pembahasan:
Kalimat‐kalimat pada paragraf terakhir ini disusun dengan pola “kalimat utama ‐>
kalimat pendukung sebab akibat (cause effect)”. Kalimat utamanya menyebutkan
bahwa dalam Bahasa Jepang, kata “semut” bermakna “serangga yang setia”. Kalimat
utama ini kemudian didukung oleh kalimat yang menyebutkan bahwa semut
sangatlah setia kepada sesama semut dan mereka jarang sekali menunjukkan perilaku
yang hanya mementingkan diri mereka sendiri (sebab). Oleh karena hal itu, manusia
selalu terkagum‐ kagum akan kesetiaan yang dimiliki oleh semut (akibat).

60. Which of the following statements about soldier ants is NOT true?
A. People can use them to close wounds.
B. They can see nothing a head of them.
C. They have powerful bites.
D. They lay many eggs.
E. They link their legs together to form a living bridge.
Jawaban: D
Pembahasan:
Pernyataan (D) yang menyatakan bahwa pasukan semut bertelur (lay egg) merupakan
pernyataan yang tidak tepat. Pada paragraf ke‐3, kalimat terakhir, disebutkan bahwa
ratu semutlah (queen) yang bertelur dan pasukan semut bertugas untuk berjaga‐jaga

38
SOAL-SOAL LITERASI BAHASA INGGRIS

selama proses tersebut.

The following text for answer the questions number 61 – 63!

BTS or Bangtan Sonyeondan is a seven-member South Korean boy band that began
formation in 2010 and debuted in 2013 under Big Hit Entertainment. The members’ names
are Jin, Suga, J-Hope, RM, Jimin, V, and Jungkook. This group was originally a hip hop
group, but their musical style has evolved to include a wide range of genres. Their lyrics,
often focused on personal and social commentary, touch on the themes of mental health,
troubles of school-age youth, loss, the journey towards loving oneself, and individualism.
Their work features references to literature and psychological concepts and includes an
alternative universe storyline. The group has released several albums and performed on
several world tours.
They debuted on June 13, 2013 with the single "No More Dream" from their first album,
2 Cool 4 Skool. BTS has won several awards in the 'New Artist of the Year' category,
including the 2013 MelOn Music Awards, the 2013 Golden Disk Awards and the 2014 Seoul
Music Awards. A year after their debut, they received bonsang awards for Dark & Wild and
The Most Beautiful Moment In Life, Part 1. Known for their large international fan base and
having performed in Asia, North America and South America, their brilliant performance
won the 'Best World Performer' award at Mnet Asian Music Awards 2015.
Their fourth mini album, The Most Beautiful Moment In Life, Part 2 was able to occupy
the top position of the Billboard World Albums Chart and was able to stay there for several
weeks, making BTS the first Korean musician to achieve this achievement. BTS received a
lot of praise for their activity. on social media and was named by Forbes as the artist with the
most retweets on Twitter in March 2016. Following this, Twitter launched emojis for BTS for
a duration of one month. The appearance of this emoji makes BTS the first K-Pop group to
have their own emoji

61. The genre of the text is …


A. Descrptive text
B. Narrative text
C. Report text
D. New item

39
SOAL-SOAL LITERASI BAHASA INGGRIS

E. Explanation text
Jawaban: A

62. How many member of BTS?


A. Nine member
B. Seven member
C. Five member
D. For member
E. Ten member
Jawaban: B

63. When they debuted with the single “ No more dream “?


A. On August 13, 2013
B. On September 14,2015
C. On August 14, 2014
D. On June 14, 2014
E. On June 13, 2013
Jawaban: E

The question is based on the following passage.

    Supply chain weaknesses were brought to the forefront during the COVID-19 pandemic,
especially for industries relying on electronics, as the flow of raw materials slowed or
sometimes stopped. On top of that, shifting consumer values and tougher environmental
regulations have resulted in more people buying hybrid vehicles. The batteries in these cars
require rare metals that, depending on their supplies, can have volatile and unpredictable
prices. However, there are other scarce elements and materials that may be used in smaller
amounts in hybrid models versus conventional gas vehicles, raising the question of how these
vehicles really compare with regard to supply chain vulnerabilities. Randolph Kircherian and
colleagues wanted to develop a comprehensive comparison of the elements and compounds
that go into all the parts in gas-powered, self-charging hybrid and plug-in hybrid cars,
calculating each of the three vehicles' materials cost vulnerability.

40
SOAL-SOAL LITERASI BAHASA INGGRIS

    The researchers collected information on the compounds in the more than 350,000 parts
used to build seven vehicles from the same manufacturer with different levels of
electrification, including four sedans and three sport utility vehicles (SUVs). Then, they
calculated the amount of the 76 chemical elements present, as well as a few other materials,
in each car type. To develop a monetary metric for vulnerability, the team considered the
weight of each component, along with its average price and price volatility between 1998 and
2015. The results showed that self-charging hybrid and plug-in hybrid vehicles have twice the
raw material cost risks. The largest contributors to the increase in cost risks were battery-
related elements, such as cobalt, nickel, graphite and neodymium. The researchers say that as
manufacturers ramp up electric vehicle production to meet demand, reducing raw material
cost risks with long-term supplier contracts, substituting some materials or recycling others
will be a good idea.

64. The author would apparently agree that the relationship between environmental
regulations and the purchase of hybrid vehicles in paragraph 1 is similar to the
phenomenon of....
A. cars and traffic 
B. oil and gas stations
C. people and vehicles
D. gasoline and transportation 
E. modern life and electric cars

Jawaban: A
Pembahasan:

Soal tersebut menanyakan analogi yang tepat dari hubungan antara peraturan lingkungan
dan pembelian mobil hibrida.

On top of that, shifting consumer values and tougher environmental regulations have
resulted in more people buying hybrid vehicles

Informasi yang berkaitan dengan hal tersebut terdapat di paragraf pertama kalimat kedua.
Pada kalimat tersebut, dikatakan bahwa peraturan lingkungan menyebabkan masyarakat
membeli mobil hibrida. Analogi yang digunakan di sini adalah analogi sebab akibat.
Dengan demikian, analogi yang tepat adalah antara cars dan traffic karena mobil

41
SOAL-SOAL LITERASI BAHASA INGGRIS

menyebabkan kemacetan sehingga pilihan A tepat.

Pilihan B (minyak dan tempat pengisian bahan bakar) tidak tepat. Minyak tidak
menyebabkan adanya pom bensin. Pom bensin didirikan karena adanya kendaraan.

Pilihan C (orang dan kendaraan) tidak tepat. Penyebab adanya kendaraan adalah jarak
tempuh, bukan manusia.

Pilihan D (bahan bakar minyak dan transportasi) tidak tepat. Kendaraanlah yang


menyebabkan adanya bensin, bukan sebaliknya.

Pilihan E (kehidupan modern dan mobil listrik) tidak tepat. Kehidupan modern tidak
menyebabkan adanya mobil listrik. Mobil listrik diciptakan karena adanya masalah
lingkungan.
Jadi, pilihan jawaban yang tepat adalah A.

The question is based on the following text.

    Exercise is painful. As the cliché goes, “No pain, no gain.” When the body exerts itself,
pumping action out of muscles to tear them down and build their mass, it’s left with a
soreness. Doctors, coaches, and mothers all recommend heat for tense sore muscles—warm
baths, moist towels, hot-water bottles, or heated pads as thermotherapy techniques. But how
exactly does applying this heat help the pain and relax the muscles?

    While exercising, the body requires more energy than it can produce through aerobic
respiration, or the intake of oxygen. To create enough energy for vigorous movement, the
body goes through another process: anaerobic respiration. This type of energy production
burns sugars without oxygen, producing lactic acid within exerted muscles. Overworked
muscles and a buildup of lactic acid are what cause the pain associated with exercising. When
heat is applied to a sore area of the body, blood vessels widen and blood flow increases to
transport excess lactic acid and other toxins away from tired muscles. These muscles are also
made more elastic by the heat, and nerve endings are stimulated to block pain signals. 

65. What is the purpose of the underlined sentence in paragraph 1?

42
SOAL-SOAL LITERASI BAHASA INGGRIS

A. highlight the process of muscle building during exercise


B. illustrate the cliché related to exercise and pain
C. explain the sources of pain while exercising 
D. show the process of soreness on muscles 
E. define the cliché “No pain, no gain”   

Jawaban: B

Pembahasan:
Soal menanyakan tujuan penulisan kalimat bergaris bawah yang terdapat pada
paragraf pertama.

Terjemahan kalimat tersebut adalah ketika tubuh berolah raga, menuntut otot untuk
bekerja keras dan mambangun massa, tubuh akan terasa nyeri. Dengan kata lain,
kalimat tersebut bertujuan mengilustrasikan ungkapan “No pain, no gain,” yang
berarti 'Tidak ada pengorbanan, tidak ada hasil.’ (Pilihan B). Pengorbanan yang
dimaksud diilustrasikan dengan rasa nyeri pada tubuh dan hasil diilustrasikan
dengan pembentukan massa otot. 

Pilihan A tidak tepat. Kalimat tersebut tidak menyoroti proses otot terbentuk saja,
melainkan juga efek yang ditimbulkannya, yaitu rasa nyeri.

Pilihan C tidak tepat. Kalimat tersebut menyebutkan bahwa sumber rasa nyeri
pada tubuh saat berolahraga adalah kerja keras otot saja. Tidak ada penjelasan
mengenai sumber lain.

Pilihan D tidak tepat. Kalimat tersebut bukan hanya menunjukkan tentang nyeri
pada otot yang diakibatkan dari berolah raga, tetapi juga terbentuknya massa otot.

Pilihan E tidak tepat. Kalimat tersebut tidak memuat definisi dari ungkapan “No
pain, no gain”, melainkan hanya memberikan ilustrasi untuk ungkapan tersebut. 
Oleh karena itu, pilihan jawaban yang tepat adalah B.

Read the text below to answer the question!

43
SOAL-SOAL LITERASI BAHASA INGGRIS

    A team led by UC Riverside geologists has discovered the first ancestor on the family tree
that contains most familiar animals today, including humans. The tiny, wormlike creature,
named Ikaria wariootia, is the earliest bilaterian, or organism with a front and back, two
symmetrical sides, and openings at either end connected by a gut. The paper is published
today in Proceedings of the National Academy of Sciences.

    For 15 years, scientists agreed that fossilized burrows found in 555 million-year-old
Ediacaran Period deposits in Nilpena, South Australia, were made by bilaterians. However,
there was no sign of the creature that made the burrows, leaving scientists with nothing but
speculation. Then, Scott Evans, a recent doctoral graduate from UC Riverside; and Mary
Droser, a professor of geology, noticed miniscule, oval impressions near some of these
burrows. With funding from a NASA exobiology grant, they used a three-dimensional laser
scanner that revealed the regular, consistent shape of a cylindrical body with a distinct head
and tail and faintly grooved musculature. The animal ranged between 2-7 millimeters long
and about 1-2.5 millimeters wide, with the largest the size and shape of a grain of rice -- just
the right size to have made the burrows.

    "We thought these animals should have existed during this interval, but always understood
they would be difficult to recognize," Evans said. "Once we had the 3D scans, we knew that
we had made an important discovery."

66. What is the main idea of the passage?


A. the descendant on the family tree of familiar animals found in Australia 
B. a team led by UC Riverside geologists 
C. Australian geologists finding most animals
D. the small ancestor of familiar animals identified in Australia
E. the geologists’ discovery of a worm in Australia 

Jawaban: D

Pembahasan:

Soal menanyakan ide pokok yang tepat dari teks bacaan.

Pada keseluruhan paragraf, teks tersebut membahas mengenai penemuan makhluk


yang dianggap sebagai leluhur dari silsilah keluarga sebagian besar binatang yang

44
SOAL-SOAL LITERASI BAHASA INGGRIS

ada saat ini.

Pilihan A tidak tepat karena terdapat kata The descendant yang berarti 'keturunan'.


Hal tersebut merupakan antonim dari kata ancestor yang berarti 'leluhur'.

Pilihan B dan C tidak tepat karena yang menjadi fokus pada kedua pilihan tersebut
adalah tim ahli geologi yang melakukan penemuan.

Pilihan D tepat karena pernyataan tersebut sesuai dengan yang disampaikan pada


teks bacaan.

Pilihan E tidak tepat karena makhluk yang ditemukan bukanlah


seekor worm (cacing), tetapi makhluk seperti cacing . Informasinya terdapat pada
kalimat kedua paragraf pertama.

Oleh karena itu, pilihan jawaban yang tepat adalah D.

Read the following text for the question number 67!

(The story by Oiwan Lam originally appeared on Global Voices, on December 2, 2020)
    Over a dozen journalists resigned on December 1 from Hong Kong Cable Television
Limited, a major subscription-based TV station in Hong Kong, in protest after the media
company fired 40 editorial staff in what is perceived as a political crackdown. Those laid off
include the head of the China desk and the entire team of the award-winning News Lancet
program. The broadcaster cited financial reasons for the layoff, a claim the newsroom staff
rebukes. The entire China desk, a total of 12 people, has since resigned in protest, along with
the heads of the international and finance desks and the newsroom's chief assignment editor.

    The layoffs were sudden and took immediate effect — those who were sacked were told to
leave the office on the same day. In shock, members of staff gathered outside the room of the
news director demanding an explanation. Among those laid off is Wong Lai-ping, former
deputy news editor at the China desk. On a radio interview aired on December 2, she rebuked
the management's claim that the decision was made out of financial difficulties, remarking
that in August, the management fired the then-executive director Fung Tak-hung and

45
SOAL-SOAL LITERASI BAHASA INGGRIS

replaced him with four new managers. Wong said that the new management often made
editorial requests, such as live broadcasts of China's foreign minister's regular press
conference, or modifying headlines. According to Wong, the newsroom staff refused to
comply with such requests.

    The Hong Kong Journalists Association commented on the layoffs, “Given the [New
Lancet] team’s coverage of the police and administration, it’s difficult not to see this as
minimizing sensitive reporting in the name of cost-cutting”.

67. Based on the text, the trigger of this resignation en masse is the belief that ….
A. the previous layoffs were politically motivated
B. financial reasons affected the layoffs the most
C. the journalists could demand for higher salary
D. political crackdown barely prompted the layoffs
E. the management's disapproved of staffs' refusal of demands
Jawaban: A

Pembahasan:

Pada soal ini, kita harus menentukan pemikiran atau kepercayaan yang menjadi


pemicu/penyebab dari pengunduran diri massal (besar-besaran) para jurnalis tersebut.

Pada awal paragraf pertama disebutkan, “Lebih dari 12 orang jurnalis mengundurkan
diri pada 1 Desember dari Hong Kong Cable Television Limited …, sebagai
protes setelah perusahaan media tersebut memecat 40 staf editorial yang dianggap
sebagai tindakan keras politik”. Penulis melanjutkan, “... Penyiar mengutip alasan
keuangan untuk PHK tersebut, klaim yang ditegur oleh staf ruang berita”.
Selanjutnya, paragraf kedua menyebutkan, “Salah satu yang diberhentikan adalah
Wong Lai-ping, mantan wakil editor berita di China desk .., dia menegur
klaim manajemen bahwa keputusan itu diambil karena kesulitan keuangan, dengan
mengatakan bahwa pada bulan Agustus, manajemen memecat direktur eksekutif
Fung Tak-hung dan menggantinya dengan 4 manajer baru”.

Berdasarkan penjelasan tersebut, kita dapat mengetahui bahwa pengunduran diri


massal (lebih dari 12 orang) tersebut dipicu atas adanya PHK (Pemutusan Hubungan
Kerja) atau layoffs terhadap 40 staf editorial. Mereka percaya bahwa tindakan

46
SOAL-SOAL LITERASI BAHASA INGGRIS

tersebut sebenarnya bukan disebabkan oleh kesulitan keuangan, tetapi adanya unsur
politik. Dengan kata lain, mereka memiliki kepercayaan bahwa PHK tersebut
bermotif politik, maka pilihan A tepat.

The text is for the following question!

    Television was once the newest technology in our homes, and then came videos and
computers. Today’s children are growing up in a rapidly changing digital age that is far
different from their parents. A variety of technologies are all around us in our homes, offices,
and schools. When used wisely, technology and media can support learning children’s
relationships both with adults and their peers.

    Based on some evidence by research, there has never been a more important time to apply
principles of development and learning when considering the use of cutting-edge
technologies and new media as the so-called interactive media. Interactive media refers to
digital and analog materials, including software programs, applications (apps), some
children’s television programming, e-books, the Internet, and other forms of content designed
to facilitate active and creative use by young children and to encourage social engagement
with other children and adults.

When the integration of technology and interactive media in early childhood programs is
built upon solid developmental foundations, and early childhood professionals are aware of
both the challenges and the opportunities, educators are positioned to improve program
quality by intentionally leveraging the potential of technology and media for the benefit of
every child.

    This statement provides guidance for early childhood educators about the use of
technology and interactive media in ways that can optimize opportunities for young
children’s development. In this statement, the definition of technology tools encompasses a
broad range of digital devices such as computers, tablets, multi-touch screens, interactive
whiteboards, mobile devices, cameras, audio recorders, electronic toys, games, e-book
readers, and older analog devices still being used such as tape recorders, record and cassette
players, projectors, and microscopes.

47
SOAL-SOAL LITERASI BAHASA INGGRIS

    By appropriately and intentionally using the technology of his day—broadcast television—
to connect with each individual child and with parents and families, it demonstrated the
positive potential of using technology and media in ways that are grounded in principles of
child development.

68. The author holds the assumption that ....


A. any forms of content designed to facilitate creative children to learn will
challenge them 
B. interactive media are any kind of media that include hardware and artifacts of
education 
C. the integration of technology and interactive media in childhood programs leads
to improved program quality 
D. applying principles of development and learning of children is secondary in the
use of the interactive media 
E. technology and media can be used to facilitate learning and establish better
relationships with other children and adults
Jawaban: E

The text is for the following question!

Studies show that people use approximately 20% less energy when they are billed for exactly
what they use. In addition, water is treated at Dockside Green and reused on site for flushing
toilets.
(5) Planners of eco-communities such as Dockside Green must take the future into account.
Dockside Green will reuse 90% of its construction waste.
They also plan to continue using local suppliers for all of their transport and maintenance
needs. This is a great way to reduce emissions.
Dockside residents will be encouraged to make use of a mini ransit system and buy into the
community's car share program. Finally, plans are in the works for high-tech heating system
that will use renewable biomass instead of fossil fuels.

69. By saying "Today's buzz words. which include global warming and zero emissions, are
causing every day people to look for ways to reduce their carbon footprint," in lines 1-3
the author implies that?

48
SOAL-SOAL LITERASI BAHASA INGGRIS

A. Environmental issues have made people more cautious of their actions


B. Global warming and zero emissions become everyone's concern
C. People have to take care of their greenhouses carefully
D. Eco-friendly environmental programs should be strongly enforced
E. All people around the world are aware of global warming
Jawaban: B

The text is for the following question!

As a result, we have lost the opportunity to learn about these areas and the benefits and
opportuni ties they might have yielded.

Today our remaining natural areas serve important roles in the study of ecology, botany,
zoology, geology, and soil science.

They provide controls for comparison against managed or exploited resources. They also
provide educational and cultural infor mation important in the study of science, local history,
conservation, and nature studies such as bird-watching, insect study, and tree identification.

Preservation of natural areas also provides many practical benefits. For example, natural
areas contain the biological raw materials necessary for the development of products that
could greatly benefit the health and well-being of a man.

A new wonder drug or fine industrial prod uct may exist now only in some inconspicuous
organism harbored in a nature preserve. Up to approximately half of the drugs currently in
use contain derivatives of wild plants, yet only a small percentage of all plants have been
investigated for their potential in such uses.

The need to protect the remaining 98 percent of natural areas until they can be researched is
obvious.

From a genealogical standpoint, we have roots linking us to our ancestors. With so many of
our natural areas now gone, those that remain are a vital link to the past. They can help us and
future generations better understand the landscape and natural resources from which the
pioneers molded their lives.

49
SOAL-SOAL LITERASI BAHASA INGGRIS

70. The sentence “Our ancestors destroyed most of our natural areas before anyone had a
chance to study or to try to understand them” in the first paragraph can best be restated
as...
A. After we had an opportunity to deeply study our natural areas, most of them had
been destroyed by our ancestors
B. We might have had an opportunity to deeply study our natural areas if most of
them had not been destroyed by our ancestors
C. We had an opportunity to deeply study our natural areas although most of them
had been destroyed by our ancestors
D. Most of our natural areas had been destroyed by our ancestors, besides, we had an
opportunity to deeply study them
E. Before most of our natural areas had been de stroyed by our ancestors, we had an
opportunity to deeply study them
Jawaban: B

Read the text carefully


(1) In addition to being pleasant and reducing thirst, drinking tea turns out to be also
beneficial for health. Some research results show that drinking tea is beneficial for health
if the amount of tea consumed is not excessive.
(2) Female First writes that a new study in the United Kingdom shows that drinking tea
regularly can help reduce the risk of developing breast cancer. To analyze the
relationship between tea consumption and the reduced risk of breast cancer, researchers
conducted 39 studies of the benefits of tea consumption in 13,204 breast cancer
patients.Based on the results of these studies, antioxidant properties in tea can help
reduce the risk of breast cancer. Taking tea regularly causes a 21% reduction in breast
cancer risk.
(3) According to a female health specialist, Dr. Catherine Hood from "The Tea Advisory
Panel (TAP)" to Female First, this finding is related to the level of polyphenols in tea.
Tea is rich in polyphenols, including catechins and gallocatchins, which have been
known to function as antioxidants and to have anti-tumor effects. The latest research
conducted by Dr. Tim Bond from The Tea Advisory Panel (TAP) shows that consuming

50
SOAL-SOAL LITERASI BAHASA INGGRIS

black tea lowers cardiovascular risks. Flavonoid in tea helps improve blood vessel
functions.
(4) In this study, 20 healthy people consumed tea three times a day for one week. The results
show that the blood vessel function of these people increases after consuming tea that is
not brewed with hot water. According to the Daily Mail, scientist from Taiwan reported
that consuming three cups of tea a day helps prevent heart disease and improves blood
circulation.
(5) Drinking more than 450 ml of tea every day will reduce the risk of arteries becoming
stiff, with a reduced risk of 22%. Research conducted by the Harvard School of Public
Health, America, also shows that tea has many health benefits. Besides being able to
lower blood pressure, tea can also help prevent the ovaries and digestive system cancer.
Consuming three cups of black tea or green tea every day can help prevent strokes.
Drinking green tea, in particular, can help reduce the risk of breast, prostate, and
endometrial cancer.

71. In which paragraph does the writer emphasize the effect of drinking tea on breast
cancer?
A. 1
B. 2
C. 3
D. 4
E. 5
Jawaban: B

Answer these questions correctly!

51
SOAL-SOAL LITERASI BAHASA INGGRIS

72. What statement is NOT TRUE according to the text?


A. She was born in Portugal
B. She speaks English fluently
C. She is familiar with in Design
D. She celebrates her birthday on January 6th
E. She got her master degree from the Creative University

Jawaban: A

Questions no 73 – 74 based on the following passage!

Imagine 15 grocery bags filled with plastic trash piled up on every single yard of shoreline in
the world. That’s how much land-based plastic trash ended up in the world’s oceans in just
one year. The world generates at least 3.5 million tons of plastic and other solid waste a day,
10 times the amount a century ago, according to World Bank researchers.

The U.S. is the king of trash, producing a world-leading 250 million tons a year—roughly 4.4
pounds of trash per person per day. And yet there are a growing number of people—often
young millennial women—who are part of a zero-waste movement.

Their yearly trash output can be small enough to fit inside an eight-ounce mason jar. These
are not wannabe hippies, but people embracing a modern minimalist lifestyle. They say it
saves them money and time and enriches their lives. Kathryn Kellogg is one of those young
millennials who has downsized her trash pile—anything that hasn’t been composted or
recycled—so two years' worth literally fits inside one 16-ounce jar.

52
SOAL-SOAL LITERASI BAHASA INGGRIS

Meanwhile, the average American produces 1,500 pounds of trash a year. (Learn more about
Kellogg in the recent plastic issue of National Geographic magazine).

We also saved about $5,000 a year by purchasing fresh food instead of packaged, buying in
bulk, and making our own products like cleaners and deodorant,” says Kellogg, who lives
with her husband in a small house in Vallejo, California. Kellogg is one of several zero-waste
bloggers who share online the details of their efforts, along with practical tips and
encouragement, for others looking to embrace a zero-waste lifestyle. In three years, she has
gained 300,000 monthly readers on her blog goingzerowaste.com and on Instagram.

“I think many people are ready to cut their waste,” says Kellogg. However, she doesn’t want
people to fixate on trying to stuff all their trash into a jar. Zero- waste is really about trying
to minimize your trash and making better choices in your life, she says. “Just do the best
you can and buy less.”

73. What is the main idea of the passage?


A. Zero-waste management movement by millennial women
B. Two years’ worth of trash in only 16-ounce jar
C. Kellogg as one of the zero-waste bloggers
D. Amount of plastic trash in the ocean
E. The U.S. as the king of trash

Jawaban : A
Pembahasan: Soal di atas menanyakan gagasan utama teks. Pilihan jawaban tepat
adalah A, karena zero-waste management movement by millennial women
mendominasi bahasan dalam paragraf.

74. What is the definition of zero-waste as stated by Kellogg?


A. Producing no waste at all
B. Make new choices in life
C. Do your best to escalate shopping
D. Trying to put all their trash into a jar
E. Make better choices to diminish waste production

Jawaban : E

53
SOAL-SOAL LITERASI BAHASA INGGRIS

Pembahasan : Soal di atas menanyakan defenisi dari zero waste menurut Kellogg yg
dapat kita temukan di paragraf terakhir, dan di pilihan jawaban pendapat Kellogg
disampaikan dalam bentuk simpulan.

The question is based on the following passage.

TikTok is often applauded for its recommendation system; once it’s finely tuned, the app
becomes one of the best scrolling experiences. My personal theory is that’s why TikTok is so
addicting—everything is so perfectly curated to your specific interests, it’s hard to put the
phone down once you’re sucked in. However, TikTok’s recommendation algorithm still has
its own flaws that the company brings up in its new blog post.
“One of the inherent challenges with recommendation engines is that they can inadvertently
limit user experience–what is sometimes referred to as a ‘filter bubble’,” the post reads. It
states that by optimizing for personalization and relevance, there is a risk of presenting an
increasingly homogenous stream of videos.
Another issue that TikTok takes seriously is not surfacing dangerous content. This is an issue
that YouTube in particular has faced criticism over for many years. According to TikTok,
content that has graphic material like medical procedures or “legal consumption of regulated
goods,” like alcohol, may not be eligible for recommendation because it could come across as
“shocking if surfaced as a recommended video to a general audience”. That’s why many
creators on TikTok will upload a video more than once or talk openly about feeling shadow
banned over particular content.

75. The writer of the passage intends to ...


A. illustrate TikTok users' concerns about the app's recommendation system
B. oppose the idea that TikTok has a sophisticated recommendation system
C. inform about the downsides of TikTok's recommendation system
D. warn the readers about TikTok’s poor recommendation system
E. explain how TikTok resolves the recommendation system issue

Jawaban : C
Pembahasan: Soal di atas menanyakan tentang tujuan dari penulis. Dalam paragraf
pertama penulis telah menyebutkan bahwa aplikasi TikTok masih memiliki
kekurangan. Di paragraf kedua penulis menyebutkan kekurangan tersebut, yaitu

54
SOAL-SOAL LITERASI BAHASA INGGRIS

membatasi pengalaman pengguna. Di paragraf selanjutnya penulis juga masih


membahas kekurangan lainnya. Jadi, dapat disimpulkan bahwa tujuan penulis adalah
membahas kekurangan-kekurangan sistem rekomendasi TikTok.

76. According to the passage, it can be predicted that ...


A. Video creators on TikTok will avoid creating dangerous content.
B. TikTok will be considered a safer social media platform compared to YouTube.
C. Videos with sensitive content can only be recommended for a specific scope of
viewers.
D. Other social media platforms will look up to TikTok for its good recommendation
system.
E. Users will be exposed to topics outside their preferences in their recommendation
feeds.
Jawaban : A
Pembahasan : Soal menanyakan prediksi yang dapat dibuat dari teks bacaan. Pilihan
A adalah yang paling tepat, karena pernyataannya berhubungan logis dengan
informasi yang disampaikan dalam paragraf ketiga, yaitu bahwa TikTok tidak akan
merekomendasikan video yang mengandung konten yang melanggar kebijakan
TikTok.

Questions no 77 – 78 based the following passage!

(The story by Sanjib Chaudhary originally appeared on Global Voices on August 8, 2020)

After news of the existence of a yellow turtle in India's eastern state of Odisha went
viral this past July, Nepalis wanted to remind the world that they made a similar discovery
first — on April 14, 2018, to be exact — when a rare, golden turtle — but a different species
— was found for the first time in southeastern Nepal’s Dhanushadham municipality.
According to a research paper by Kamal Devkota, Dev Narayan Mandal and Hinrich Kaiser,
which was published in the journal Herpetology Notes, the turtle was released into its natural
habitat after pictures were taken as proof.

A normal Indian flapshell turtle is greenish-grey in colour, with yellow marks on its
head and neck. It has a grey carapace dotted with dark yellow spots and derives its name from

55
SOAL-SOAL LITERASI BAHASA INGGRIS

the femoral flaps at the plastron, the ventral part of the shell. These turtles are found in
Bangladesh, India, Myanmar, Nepal, Pakistan and Sri Lanka, most typically in ditches, lakes,
ponds, and paddy fields with stagnant water. Omnivorous in nature, they eat anything from
leaves and flowers to snails, fish, and frogs. While a normal-coloured flapshell turtle can
easily camouflage itself in the murky, greenish water, its golden-coloured variant — a turtle
with chromatic leucism — is easily recognisable and therefore more vulnerable. Its luminous
golden colour, in particular, makes it a prized pet.

The congenital disorder of albinism creates a complete absence of pigmentation in the


skin, hair, and eyes due to the lack of tyrosinase, an enzyme involved in the production of
melanin. Researchers estimate that albinism occurs once in every 10,000 mammal births.
Leucism, on the other hand, is an extremely rare genetic condition in which animals have
reduced pigmentation. Most leucistic animals have normal-coloured eyes, whereas those with
albinism tend to have red or pink eyes.

Nepal is home to 16 species of turtles, of which four are critically endangered: the
three-striped roofed turtle (Kachuga dhongoka), the red-crowned roofed turtle (Kachuga
kachuga), the Indian narrow-headed softshell turtle (Chitra indica), and the elongated tortoise
(Indotestudo elongata). Though turtles play an important role in reducing pollution by
feeding on insects, vegetation, and dead animals, factors like habitat loss, fragmentation, and
degradation have been threatening their survival. According to “Turtles of Nepal – A Field
Guide for Species Accounts and Distribution,” the draining of wetlands for irrigation and
fishing, the excessive use of agrochemicals which eventually seep into waterways, and
unsustainable fishing methods all negatively impact turtle populations.

77. The third paragraph of the passage above mainly discusses ….


A. physical appearance of turtles
B. congenital disorders in sea-mammals
C. albinism, leucism, and how to treat them
D. the difference between two congenital illnesses
E. albinism and leucism as hazardous congenital disorders
Jawaban: D
Pembahasan : Pada paragraf ketiga penulis membahas mengenai penyakit Albinism
dan leucism pada mamalia. Pada kalimat terakhir paragraf terdapat kata “whereas”

56
SOAL-SOAL LITERASI BAHASA INGGRIS

yang menunjukkan perbedaan albinisme dan leukisme.

78. Which of the following is NOT TRUE according to the passage above?
A. A quarter of all turtle species in Nepal are threatened with extinction.
B. Both Nepalis and Indians found new rare turtles at different times.
C. A turtle with chromatic leucism is insusceptible to predators.
D. Animals with albinism and leucism can bedistinguished.
E. Turtles can help humans to keep the environment healthy.

Jawaban : C
Pembahasan : (Seekor penyu dengan chromatic leucism tidak rentan menjadi
mangsa) tidak sesuai atau bertentangan dengan pernyataan “... its golden-coloured
variant — a turtle with chromatic leucism — is easily recognisable and therefore
more vulnerable.” (... variannya yang berwarna keemasan, penyu dengan chromatic
leucism mudah dikenali dan karena itulah lebih rentan/mudah diserang.). Keadaan
mudah diserang ini menyiratkan bahwa penyu tersebut juga akan mudah dijadikan
mangsa oleh predator

Questions 79 to 81 based on the following text!

Sleepiness after eating is a response of the body to chemical changes during the digestion
process. This is normal and it happens to everybody. However, if sleepiness occurs every
time after eating and obstructs your ability to function, this may be a concern. Sleepiness
after eating is caused by many factors, such as the type of food you consume, messy sleeping
habits, your health condition and so forth. The body requires energy to function and this
energy is obtained through food. Post-eating, the body releases hormones such as amylin,
glucagon and cholecystokinin. These hormones increase blood sugar levels, creating a feeling
of fullness and producing insulin that will be streamed through cell tissues and provide
energy for them. At the same time, the brain releases serotonin that causes drowsiness.

Moreover, food also influences melatonin production in the brain. This is the hormone that is
responsible for sleepiness post-meal. Some foods that are rich in protein, such as meat,

57
SOAL-SOAL LITERASI BAHASA INGGRIS

poultry, eggs, fish, spinach, tofu, cheese and soybeans can trigger more drowsiness than
others. Additionally, foods containing carbohydrates also help produce serotonin and
tryptophan amino acids found in the brain. This is the reason why you feel sleepy after eating
carbohydrate-rich foods.

Overeating can also cause sleepiness. Post-meal, the body streams more blood to the
digestive system to better digest foods in massive amounts. This causes a temporary blood
and nutrients shortage in the brain. To prevent post-meal drowsiness, it’s better to eat a
balanced diet containing vegetables, grains and good fats to provide continuous energy. Drink
lots of water and limit your sugar intake.

Bad sleeping patterns can also cause sleepiness after eating. After a meal, the body feels full
and relaxed, making the body feel like it is resting, resulting in a feeling of sleepiness,
particularly if you didn’t get a good night’s sleep the night before. To avoid this, improve
your sleeping habits to prevent stress. Engaging in regular physical exercise can help you get
a good night’s sleep. It is recommended that you avoid napping if you are having trouble
sleeping at night.

79. What is the author’s purpose in writing the passage?


A. To discuss the amount of food to consume so as not to feel sleepy
B. To inform the readers about the factors of sleepiness after eating
C. To argue the effect of eating too much foods for the brain
D. To investigate what causes drowsiness after overeating
E. To tell the readers the benefit of eating certain foods

Jawaban : B
Pembahasan : Adapun tujuan penulis pada teks di atas adalah untuk
menginformasikan penyebab kantuk setelah makan.

80. By writing the sentences in paragraph 2, the author intends to tell the readers about….
A. foods that are rich in protein, such as meat, eggs, fish, tofu, cheese and soybeans
can generate more drowsiness
B. overeating protein and carbohydrate-rich foods obstructs your brain’s ability to
function well

58
SOAL-SOAL LITERASI BAHASA INGGRIS

C. the reason why carbohydrate-rich foods make you feel sleepy


D. food combinations containing tryptophan amino acid and carbohydrates make
you feel drowsy
E. the hormone melatonin production in the brain is responsible for sleepiness after
eating
Jawaban: D

Pembahasan:
Paragraf ke-2 penulis intinya bermaksud untuk memberitahu pembaca bahwa
kombinasi makanan yang mengandung asam amino triptofan (protein) dan
karbohidrat membuat Anda merasa mengantuk (Additionally, foods containing
carbohydrates also help produce serotonin and tryptophan amino acids found in the
brain. This is the reason why you feel sleepy after eating carbohydrate-rich foods.).
Maka pilihan jawaban (D) food combinations containing tryptophan amino acid and
carbohydrates make you feel drowsy merupakan opsi yang tepat.

81. It can be predicted from the passage that….


A. the more fish you eat, the better it is for the brain
B. the less meat you eat, the more you will feel drowsy
C. the finer you manage your sleep, the more lethargic you feel
D. the less you eat carbohydrate-rich foods, the less you will feel drowsy
E. the less you consume protein and carbohydrates, the better it is for the body

Jawaban: D
Pembahasan : Dari kalimat “This is the reason why you feel sleepy after eating
carbohydrate-rich foods.” dapat diprediksikan bahwa semakin sedikit Anda
mengonsumsi makanan yang kaya karbohidrat, maka rasa kantuk akan berkurang.
Maka pilihan jawaban (D) the less you eat carbohydrate-rich foods, the less you will
feel drowsy merupakan opsi yang tepat.

Questions 82 to 83 based on the following passage!

59
SOAL-SOAL LITERASI BAHASA INGGRIS

Indonesia is the world’s third-largest coffee producer and exporter, after Brazil and Vietnam.
National coffee output has grown over the past decades, albeit not in a linear fashion as
harvests fluctuate strongly from one year to another depending on the weather. With per-
capita coffee consumption on the rise both in Indonesia and the wider region, there is obvious
room for further growth, but there is also an obvious need for investment. The capital
required to take Indonesia’s coffee industry to the next level presents appealing prospects for
investors, while the country’s burgeoning coffee culture also brings opportunities for foreign
exporters.

Indonesia’s tropical climate produces almost ideal conditions for planting coffee. Today,
most Indonesian coffee comes from Sumatra, but Sulawesi and Kalimantan, the Lesser Sunda
Islands of Bali, Sumbawa and Flores as well as the country’s easternmost region of Papua all
contribute to national output. Robusta coffee makes up more than three quarters of
Indonesia’s produce; the remainder is of the milder Arabica type. The numerous coffee-
growing regions in the country produce beans of distinct flavors and properties, and a number
of highland Arabica coffees from Indonesia are recognized by aficionados the world over.

Indonesian coffee exports rose from 336,840 tonnes (or 5,614,000 60-KG bags) in crop year
2000/2001 to 656,400 tonnes (10,940,000 bags) in 2012/2013, according to data collated by
the International Coffee Organization. Total production over the same period increased from
419,220 tonnes to 763,800 tonnes. At present, the principal destinations for Indonesian coffee
are the US, Japan and Western Europe (particularly Germany), but Indonesia is well placed to
capitalize on the fast-rising demand in the ASEAN region and in China.

Indonesian per-capita consumption of around 1.2 kg in 2012 pales against more than 4 kg in
the US, around 7 kg in the world’s number one coffee producer Brazil and more than 10 kg in
various European countries. But with Indonesian per-capita consumption having already
doubled in just a few years, domestic demand looks to be on a fast growth trend. This puts the
world’s fourth-most populous country on course to become a leading coffee market. Local
demand is driven by the lifestyle changes that accompany urbanization and economic
development. Caffeine consumption tends to increase when a larger part of the labor force
works in an office environment.

82. What is the best summary of the passage?

60
SOAL-SOAL LITERASI BAHASA INGGRIS

A. The primary destinations for Indonesian coffee export are the US, Japan and
Western Europe
B. Indonesia is well placed to capitalize on the fast-rising demand in the ASEAN
region
C. Indonesia’s coffee growing and Indonesia’s coffee industry needs growth capital
D. The various coffee-growing in the country produce beans of distinct flavors
E. Total production of Indonesian coffee over the same period has increased

Jawaban: C
Pembahasan : Ringkasan yang tepat dari teks di atas adalah pilihan jawaban (C)
Indonesia’s coffee growing and Indonesia’s coffee industry needs growth capital
yang bermakna “Perkembangan kopi Indonesia dan industri kopi Indonesia perlu
pertumbuhan modal” karena kalimat ini mewakili isi dari teks di atas sebagaimana
yang tercantum di kalimat utama paragraf pertama “Indonesia is the world’s third-
largest coffee producer and exporter, after Brazil and Vietnam.

83. What can be inferred from paragraph 3 in the passage?


A. The coffee export has decreased in several years
B. Total production deducted from 419,220 tonnes to 763,800 tonnes
C. Indonesian coffee exports increased from 2000/2001 to 2012/2013
D. Indonesia is in a good site to take advantage of fast-rising demand in ASEAN
E. The US, Japan and Western Europe are the destination for Indonesian coffee
Jawaban: C
Pembahasan : Dari paragraf ke-3 bisa disimpulkan bahwa ekspor kopi Indonesia
telah meningkat dari tahun 2000/2001 sampai tahun 2012/2013. Ini ada pada awal
kalimat paragraf ke-3 tersebut. Maka pilihan jawaban (C) Indonesian coffee exports
increased from 2000/2001 to 2012/2013 merupakan opsi yang tepat.

The questions 84 to 85 are based on the following text.


Feline chlamydial conjunctivitis is an infection caused by a bacterial organism (called
Chlamydophila felis). The most common signs of chlamydia in cats involve the eyes or the
upper respiratory tract (nose or throat), and only when infection is not treated does it spread

61
SOAL-SOAL LITERASI BAHASA INGGRIS

to the lungs. Because chlamydia lives inside cells of the body and is not able to survive for
long in the environment, spread of infection relies on direct or close contact with an infected
cat. Following infection, the incubation period (the time between infection and development
of clinical signs of disease) is between three and ten days.
The bacteria primarily infects the conjunctiva, which are the delicate membranes lining the
eyelids and covering the edges of the eyeballs. The infection causes inflammation known as
conjunctivitis. In normal cats, the conjunctiva is not readily visible and has a pale, salmon
pink color. In cats with conjunctivitis, the conjunctiva becomes swollen and red, making it
more visible. The nictitating membrane or third eyelid in the inner corner of the eye may
protrude partially across the eye. One or both eyes may be involved.
Affected cats initially develop a watery discharge from the eyes that quickly becomes thicker
and is usually a yellow or greenish color. The eyes are uncomfortable and cats often keep the
affected eye(s) closed. Many cats remain bright and otherwise appear normal, but some may
develop a fever or lose their appetite. After one or two days, sniffles and sneezing may also
occur. In kittens, the infection may spread to the lungs and cause fatal pneumonia.

84. What does the text mainly deal with?


A. the process of chlamydial affliction in cats
B. an overview of feline chlamydial conjunctivitis
C. clinical signs of feline chlamydial conjunctivitis
D. the symptoms of feline chlamydial conjunctivitis
E. treatments for cats with chlamydia conjunctivitis
Jawaban: B
Pembahasan : Soal di atas menanyakan gagasan utama teks. Pilihan jawaban tepat
adalah B, karena an overview of feline chlamydial conjunctivitis mendominasi
bahasan dalam paragraf.

85. One of the symptoms of feline chlamydial conjunctivitis is ….


A. eating disorder
B. laboured breathing
C. low body temperature
D. abnormal eye discharge
E. dysfunctional eye membrane

62
SOAL-SOAL LITERASI BAHASA INGGRIS

Jawaban: D
Pembahasan : Soal menanyakan salah satu gejala penyakit feline chlamydial
conjunctivitis. Informasi tersebut dapat ditemukan di paragraf kedua dan terakhir.
Kedua paragraf tersebut menjelaskan tanda-tanda kucing yang terkena penyakit.

Read the text below to answer the question!

A team led by UC Riverside geologists has discovered the first ancestor on the family
tree that contains most familiar animals today, including humans. The tiny, wormlike
creature, named Ikaria wariootia, is the earliest bilaterian, or organism with a front and
back, two symmetrical sides, and openings at either end connected by a gut. The paper is
published today in Proceedings of the National Academy of Sciences.
For 15 years, scientists agreed that fossilized burrows found in 555 million-year-old
Ediacaran Period deposits in Nilpena, South Australia, were made by bilaterians. However,
there was no sign of the creature that made the burrows, leaving scientists with nothing but
speculation. Then, Scott Evans, a recent doctoral graduate from UC Riverside; and Mary
Droser, a professor of geology, noticed miniscule, oval impressions near some of these
burrows. With funding from a NASA exobiology grant, they used a three-dimensional laser
scanner that revealed the regular, consistent shape of a cylindrical body with a distinct head
and tail and faintly grooved musculature. The animal ranged between 2-7 millimeters long
and about 1-2.5 millimeters wide, with the largest the size and shape of a grain of rice -- just
the right size to have made the burrows.
"We thought these animals should have existed during this interval, but always
understood they would be difficult to recognize," Evans said. "Once we had the 3D scans,
we knew that we had made an important discovery."

86. The purpose of the passage is ….


A. to announce how great a team of geologists is
B. to deliver the information about several ancestors
C. to advertise an ancient animal found in Australia
D. to inform the readers about the important discovery
E. to reveal the sophisticated new science technology
Jawaban: D

63
SOAL-SOAL LITERASI BAHASA INGGRIS

Pembahasan:
Soal menanyakan tujuan penulisan teks bacaan tersebut.

Teks tersebut memberikan informasi mengenai penemuan Ikaria wariootia, nenek


moyang dari sebagian besar binatang yang ada saat ini yang dianggap sebagai sebuah
penemuan yang penting. Dengan demikian, tujuan teks tersebut adalah memberikan
informasi kepada pembaca mengenai penemuan penting sehingga pilihan D tepat.
Pilihan A (untuk mengumumkan betapa hebatnya sebuah tim dari ahli geologi) tidak
tepat. Fokus dari teks tersebut adalah penemuan sebuah tim geologi, yaitu seekor
bilateria, bukan para penemunya.
Pilihan B (untuk menyampaikan informasi mengenai beberapa nenek moyang) tidak
tepat. Teks tersebut tidak menyajikan informasi tentang beberapa nenek moyang,
tetapi hanya nenek moyang dari sebagian besar binatang yang ada saat ini.

Pilihan C (untuk mengiklankan seekor binatang purba yang ditemukan di Australia)


tidak tepat. Tidak ada informasi pada teks yang mengatakan bahwa penemuan di
Australia tersebut akan diiklankan.
Pilihan E (untuk mengungkapkan sebuah teknologi ilmu pengetahuan baru yang
canggih) tidak tepat. Teks tersebut berfokus pada penemuan bilateria pertama,
ikaria wariootia, oleh sebuah tim geologis, bukan tentang teknologi ilmu
pengetahuan baru.
Jadi, pilihan jawaban yang tepat adalah D.

The text is for the following question.

    Television was once the newest technology in our homes, and then came videos and
computers. Today’s children are growing up in a rapidly changing digital age that is far
different from their parents. A variety of technologies are all around us in our homes, offices,
and schools. When used wisely, technology and media can support learning children’s
relationships both with adults and their peers.

    Based on some evidence by research, there has never been a more important time to apply
principles of development and learning when considering the use of cutting-edge
technologies and new media as the so-called interactive media. Interactive media refers to

64
SOAL-SOAL LITERASI BAHASA INGGRIS

digital and analog materials, including software programs, applications (apps), some
children’s television programming, e-books, the Internet, and other forms of content designed
to facilitate active and creative use by young children and to encourage social engagement
with other children and adults.

    When the integration of technology and interactive media in early childhood programs is
built upon solid developmental foundations, and early childhood professionals are aware of
both the challenges and the opportunities, educators are positioned to improve program
quality by intentionally leveraging the potential of technology and media for the benefit of
every child.

    This statement provides guidance for early childhood educators about the use of
technology and interactive media in ways that can optimize opportunities for young
children’s development. In this statement, the definition of technology tools encompasses a
broad range of digital devices such as computers, tablets, multi-touch screens, interactive
whiteboards, mobile devices, cameras, audio recorders, electronic toys, games, e-book
readers, and older analog devices still being used such as tape recorders, record and cassette
players, projectors, and microscopes.

    By appropriately and intentionally using the technology of his day—broadcast television—
to connect with each individual child and with parents and families, it demonstrated the
positive potential of using technology and media in ways that are grounded in principles of
child development.

87. How does the second sentence relate to the first sentence in paragraph 2?
A. It shows the importance of interactive media to children’s lives. 
B. It argues that interactive media include digital materials only.
C. It explains the definition and examples of interactive media.
D. It states that the children and adults nowadays are familiar with digital
technology.
E. It shows that interactive media can support social engagement among adults.

Jawaban: C

65
SOAL-SOAL LITERASI BAHASA INGGRIS

Pembahasan:

Soal tersebut menanyakan hubungan kalimat pertama dan kedua pada paragraf
kedua.

Kalimat pertama menjelaskan pentingnya menerapkan aturan-aturan pembelajaran


ketika mulai menggunakan teknologi baru dalam pembelajaran, yaitu media
interaktif. Kalimat kedua menjelaskan bahwa media interaktif mengacu pada materi
digital dan analog, termasuk program perangkat lunak, aplikasi, beberapa program
televisi anak-anak, e-book, internet, dan bentuk konten lain yang dirancang untuk
memfasilitasi penggunaan aktif dan kreatif oleh anak-anak muda dan untuk
mendorong keterlibatan sosial dengan anak-anak lain dan orang dewasa.

Berdasarkan hal tersebut, dapat disimpulkan bahwa kalimat kedua menjelaskan


definisi dan contoh media interaktif yang disebutkan di kalimat pertama.

Jadi, pilihan jawaban yang tepat adalah C.

The text is for the following question.

Television was once the newest technology in our homes, and then came videos and
computers. Today’s children are growing up in a rapidly changing digital age that is far
different from their parents. A variety of technologies are all around us in our homes, offices,
and schools. When used wisely, technology and media can support learning children’s
relationships both with adults and their peers.

    Based on some evidence by research, there has never been a more important time to apply
principles of development and learning when considering the use of cutting-edge
technologies and new media as the so-called interactive media. Interactive media refers to
digital and analog materials, including software programs, applications (apps), some
children’s television programming, e-books, the Internet, and other forms of content designed
to facilitate active and creative use by young children and to encourage social engagement
with other children and adults.

66
SOAL-SOAL LITERASI BAHASA INGGRIS

    When the integration of technology and interactive media in early childhood programs is
built upon solid developmental foundations, and early childhood professionals are aware of
both the challenges and the opportunities, educators are positioned to improve program
quality by intentionally leveraging the potential of technology and media for the benefit of
every child.

    This statement provides guidance for early childhood educators about the use of
technology and interactive media in ways that can optimize opportunities for young
children’s development. In this statement, the definition of technology tools encompasses a
broad range of digital devices such as computers, tablets, multi-touch screens, interactive
whiteboards, mobile devices, cameras, audio recorders, electronic toys, games, e-book
readers, and older analog devices still being used such as tape recorders, record and cassette
players, projectors, and microscopes.

    By appropriately and intentionally using the technology of his day—broadcast television—
to connect with each individual child and with parents and families, it demonstrated the
positive potential of using technology and media in ways that are grounded in principles of
child development.

88. The author would apparently agree on the idea that ...  
A. digital media can harm learning and relationships 
B. technology provides children with enjoyable experiences
C. technology may bring more good than harm when it is used wisely 
D. technology can influence the relationships between children and their friends
E. technology minimizes the potential for children’s learning and engagement with
others
Jawaban: C
Pembahasan:
Soal tersebut menanyakan kesimpulan tersirat penulis menurut teks tersebut.
Berdasarkan paragraf ketiga (educators are positioned to improve program quality
by intentionally leveraging the potential of technology and media for the benefit of
every child), paragraf keempat (the use of technology and interactive media in ways
that can optimize opportunities for young children’s development), dan paragraf
kelima (it is demonstrated the positive potential of using technology and media in

67
SOAL-SOAL LITERASI BAHASA INGGRIS

ways that are grounded in principles of child development), dapat disimpulkan


bahwa penulis mengungkapkan tentang penggunaan teknologi yang jika digunakan
secara bijak, dapat membantu dan mengoptimalkan perkembangan anak.
Dengan demikian, kesimpulan tersirat yang tepat adalah teknologi membawa lebih
banyak kebaikan daripada kerugian jika digunakan secara bijak.
Jadi, pilihan jawaban yang tepat adalah C.

The text is for the following question.

    Television was once the newest technology in our homes, and then came videos and
computers. Today’s children are growing up in a rapidly changing digital age that is far
different from their parents. A variety of technologies are all around us in our homes, offices,
and schools. When used wisely, technology and media can support learning children’s
relationships both with adults and their peers.

    Based on some evidence by research, there has never been a more important time to apply
principles of development and learning when considering the use of cutting-edge
technologies and new media as the so-called interactive media. Interactive media refers to
digital and analog materials, including software programs, applications (apps), some
children’s television programming, e-books, the Internet, and other forms of content designed
to facilitate active and creative use by young children and to encourage social engagement
with other children and adults.

    When the integration of technology and interactive media in early childhood programs is
built upon solid developmental foundations, and early childhood professionals are aware of
both the challenges and the opportunities, educators are positioned to improve program
quality by intentionally leveraging the potential of technology and media for the benefit of
every child.

    This statement provides guidance for early childhood educators about the use of
technology and interactive media in ways that can optimize opportunities for young
children’s development. In this statement, the definition of technology tools encompasses a
broad range of digital devices such as computers, tablets, multi-touch screens, interactive

68
SOAL-SOAL LITERASI BAHASA INGGRIS

whiteboards, mobile devices, cameras, audio recorders, electronic toys, games, e-book
readers, and older analog devices still being used such as tape recorders, record and cassette
players, projectors, and microscopes.

    By appropriately and intentionally using the technology of his day—broadcast television—
to connect with each individual child and with parents and families, it demonstrated the
positive potential of using technology and media in ways that are grounded in principles of
child development.

89. The author holds the assumption that ....


A. any forms of content designed to facilitate creative children to learn will
challenge them 
B. interactive media are any kind of media that include hardware and artifacts of
education 
C. the integration of technology and interactive media in childhood programs leads
to improved program quality 
D. applying principles of development and learning of children is secondary in the
use of the interactive media
E. technology and media can be used to facilitate learning and establish better
relationships with other children and adults
Jawaban: E

Pembahasan:

Soal tersebut menanyakan asumsi penulis.

Berdasarkan kalimat By appropriately and intentionally using the technology of


this day—broadcast television—to connect with each individual child and with
parents and families, it demonstrated the positive potential of using technology and
media in ways that are grounded in principles of child development, dapat
disimpulkan bahwa menggunakan teknologi dengan tepat dan ditujukan
untuk terhubung dengan setiap anak dengan orangtua dan keluarganya,
menunjukkan potensi positif penggunaan teknologi dan media dengan cara yang
didasarkan pada prinsip-prinsip perkembangan anak.

69
SOAL-SOAL LITERASI BAHASA INGGRIS

Dengan demikian, penulis berasumsi bahwa teknologi dan media dapat digunakan
untuk memfasilitasi pembelajaran dan menciptakan hubungan dengan anak-anak
lain serta orang dewasa. Oleh karena itu, pilihan E tepat.

Pilihan jawaban A, B, C, dan D tidak tepat karena informasi-informasi pada


pilihan jawaban tersebut tidak sesuai dengan informasi yang disampaikan pada teks.

Jadi, pilihan jawaban yang tepat adalah E.

The questions no. 90 – 92 based on the following passage.

    Around 2.8 million Indian students sat university examinations from the end of August to
September 13 as the country approached the grim milestone of 5-million total COVID-19
cases. Despite pleas from civil society to defer examinations, including by Swedish climate
activist Greta Thunberg, a Supreme Court ruling on August 17 gave the government the
green light to keep the original schedule for the engineer and medical colleges entrance
exams. 

    The JEE and the NEET, how university exams for those two careers are called, are India's
most competitive — they sprawl industries of their own, with preparation courses generating
millions of dollars annually. The court bench headed by Justice Ashok Bhushan said that “the
career of the students cannot be put on peril for long, and a full academic year cannot be
wasted.” The court will not hear any further petitions regarding postponing future exam
dates.

    According to the Ministry of Education, 74 percent of the students registered to the first of
the JEE took the exam, while an above 80-percent attendance was observed for the NEET
exam. Students are now gearing up for the second stage of the JEE (the JEE Advanced),
which will take place on September 27. Protests against the exam date have dwindled since
the Supreme Court decision.

90. What can be inferred from the passage?  


A. Indian students were reluctant to join university preparation courses.

70
SOAL-SOAL LITERASI BAHASA INGGRIS

B. Indian students were forced to sit the JEE and the NEET exam.
C. The JEE and the NEET are annually held by the Supreme Court.
D. The Supreme Court considered the pleas from society. 
E. The university entrance tests were finally rescheduled.
Jawaban: B

Pembahasan:

Soal menanyakan kesimpulan tersirat yang tepat berdasarkan informasi pada teks
tersebut.

Pada paragraf pertama, dijelaskan bahwa meskipun ada permohonan dari masyarakat
untuk menunda jadwal ujian masuk universitas terkait dengan banyaknya kasus
COVID-19, putusan Mahkamah Agung pada 17 Agustus memberi pemerintah lampu
hijau untuk mempertahankan jadwal asli untuk ujian masuk insinyur dan perguruan
tinggi kedokteran. Pada akhir paragraf kedua, dijelaskan bahwa pengadilan tidak akan
mempertimbangkan petisi apapun mengenai jadwal tersebut. Kemudian pada paragraf
ketiga dijelaskan bahwa ujian JEE dan NEET sudah dilaksanakan dan diikuti oleh
para mahasiswa.

Informasi pada ketiga paragraf tersebut menunjukkan bahwa sebenarnya mahasiswa


menginginkan jadwal ujian tersebut ditunda. Namun, pada akhirnya mereka tetap
mengikutinya karena pemerintah tidak bersedia menunda jadwal tersebut. 

Dengan demikian, dapat disimpulkan bahwa siswa di India mengikuti ujian JEE dan
NEET dalam keadaan terpaksa sehingga pilihan B tepat.

Pilihan A (Siswa India enggan mengikuti kursus persiapan universitas.) tidak


tepat karena pada kalimat 1 paragraf 3, dijelaskan bahwa ujian JEE dan NEET adalah
ujian yang paling kompetitif di India dan kursus persiapan untuk kedua ujian tersebut
menghasilkan jutaan dolar setiap tahunnya. Informasi tersebut menunjukkan bahwa
kebanyakan siswa India justru kemungkinan besar mengikuti kursus persiapan untuk
menghadapi JEE dan NEET.

Pilihan C (JEE dan NEET diadakan setiap tahun oleh Mahkamah Agung.) tidak
tepat karena ujian tersebut diselenggarakan oleh pemerintah India, bukan Mahkamah

71
SOAL-SOAL LITERASI BAHASA INGGRIS

Agung. Hal ini diketahui dari informasi pada akhir paragraf pertama, yaitu ‘... putusan
Mahkamah Agung pada 17 Agustus memberi pemerintah lampu hijau untuk
mempertahankan jadwal asli untuk ujian masuk fakultas teknik dan kedokteran’. Jadi,
yang membuat jadwal dan menjadi penyelenggara ujian tersebut adalah pemerintah.

Pilihan D (Mahkamah Agung mempertimbangkan permohonan dari


masyarakat.) tidak tepat karena jelas dikatakan pada kalimat terakhir paragraf
pertama bahwa Mahkamah Agung memberi izin kepada pemerintah untuk
mempertahankan jadwal asli untuk ujian masuk insinyur dan perguruan tinggi
kedokteran meskipun ada permohonan dari masyarakat untuk menunda jadwal
tersebut. 

Dengan alasan yang sama, pilihan E (Tes masuk universitas akhirnya dijadwalkan


ulang.) tidak tepat karena jadwal semula tetap dipertahankan.

Jadi, pilihan jawaban yang tepat adalah B.

91. Which statement is CORRECT based on the passage above? 


A. The first stage of the JEE Advanced was held after September 27.
B. The exams may be postponed due to the protests against the exam date.
C. NEET is an entrance university test for both medical and engineering colleges.
D. Justice Ashok Bhushan wasn’t concerned about students’ academic and future
careers.
E. Many Indians demanded rescheduling the exams because of the high rate of
COVID-19 cases.
Jawaban: E

Pembahasan:

Soal tersebut menanyakan pernyataan yang tepat berdasarkan teks.

Pilihan A (Tahap pertama the JEE Advanced diselenggarakan setelah tanggal 27


September.) tidak tepat. Kalimat “Students are now gearing up for the second
stage of the JEE (the JEE Advanced), which will take place  on September
27.” menyatakan bahwa tahap kedua berlangsung pada tanggal 27 September

72
SOAL-SOAL LITERASI BAHASA INGGRIS

sehingga tahap pertamanya pasti sebelum tanggal tersebut.

Pilihan B (Ujian mungkin ditunda karena adanya protes menentang tanggal


ujian.) tidak tepat. Pada paragraf pertama, disebutkan bahwa Mahkamah Agung
telah mengizinkan ujian berlangsung sesuai jadwal aslinya. Selanjutnya pada
paragraf kedua disebutkan “The court will not hear any further petitions regarding
postponing future exam dates. (Pengadilan tidak akan mendengar petisi lebih lanjut
tentang penundaan tanggal ujian di masa yang akan datang.)". Kalimat tersebut
menyiratkan bahwa keputusan yang telah diambil oleh Mahkamah Agung
tersebut tidak akan berubah dan diharapkan tidak akan ada lagi protes.

Pilihan C (NEET adalah ujian masuk universitas baik untuk medis maupun
teknik.) tidak tepat. Terdapat kalimat yang menyatakan “The JEE and the NEET,
how university exams for those two careers are called ....” pada teks. Karir (careers)
yang dimaksud adalah karir yang berkaitan dengan teknik dan medis (sesuai kalimat
terakhir paragraf pertama). Namun, entah JEE atau NEET merupakan ujian yang
hanya untuk salah satu dari karir atau bidang tersebut, bukan berarti JEE atau NEET
masing-masingnya memuat ujian kedua bidang tersebut.

Pilihan D (Hakim Ashok Bhushan tidak khawatir tentang akademik dan karir
siswa.) tidak tepat. Pada paragraf kedua, terdapat kalimat “The court bench headed
by Justice Ashok Bhushan said that ‘the career of the students cannot be put on peril
for long, and a full academic year cannot be wasted’. (Bangku pengadilan yang
dipimpin Hakim Ashok Bhushan mengatakan bahwa ‘karir siswa tidak bisa lama-
lama terancam, dan satu tahun akademik penuh tidak bisa disia-siakan'.)". Kalimat
tersebut menunjukkan bahwa hakim tersebut justru peduli atau khawatir dengan karir
dan akademik siswa.

Pilihan E (Banyak masyarakat India menuntut perubahan jadwal ujian karena


tingginya tingkat kasus COVID-19.) tepat. Paragraf pertama menyebutkan bahwa
siswa India tetap mengikuti ujian universitas ketika India sedang mengalami sejarah
suram 5 juta kasus COVID-19. Terlepas dari permintaan masyarakat untuk menunda
ujian, Mahkamah Agung pada 17 Agustus memutuskan untuk menjaga jadwal asli
ujian masuk tersebut. “Sejarah suram 5 juta kasus COVID-19” menunjukkan

73
SOAL-SOAL LITERASI BAHASA INGGRIS

bahwa tingkat kasus COVID-19 yang sedang terjadi itu tinggi (high rate). Lalu,


“menunda ujian” maksudnya sama dengan “mengubah jadwal ujian yang telah ada”.

Jadi, pilihan jawaban yang tepat adalah E.

92. The author’s attitude towards the civils’ protest regarding the exam schedule is ….  
A. critical
B. positive
C. objective
D. subjective 
E. informative
Jawaban: E

Pembahasan:

Soal menanyakan sikap penulis mengenai protes masyarakat tentang jadwal ujian
yang dibahas pada teks.

Penulis menjelaskan protes masyarakat tentang jadwal ujian pada paragraf pertama.
Penulis menjelaskan bahwa Mahkamah Agung memberi izin kepada pemerintah
untuk mempertahankan jadwal asli untuk ujian masuk insinyur dan perguruan tinggi
kedokteran meskipun ada permohonan dari masyarakat untuk menunda jadwal
tersebut.

Pada paragraf kedua, penulis menyebutkan dua ujian masuk universitas, yaitu JEE
dan NEET. Kemudian dilanjutkan dengan penjelasan mengenai alasan pemerintah
mempertahankan jadwal ujian yang sudah ditetapkan sejak awal dan pengadilan juga
tidak akan mempertimbangkan petisi apapun mengenai jadwal tersebut.

Pada paragraf ketiga, penulis menjelaskan tentang persentase jumlah siswa yang
sudah mengikuti ujian JEE dan NEET. Dijelaskan pula bahwa protes mengenai
jadwal ujian sudah berkurang sejak adanya putusan dari Mahkamah Agung.

Berdasarkan topik bahasan pada masing-masing paragraf, terlihat jelas bahwa penulis
menyajikan informasi mengenai protes yang dilakukan oleh masyarakat dan sikap

74
SOAL-SOAL LITERASI BAHASA INGGRIS

pemerintah terkait jadwal ujian masuk universitas. Jadi, dapat disimpulkan bahwa
sikap penulis adalah informatif sehingga pilihan E tepat.

Pilihan A tidak tepat karena tidak ada kalimat berupa kritikan penulis terhadap
protes masyarakat tentang jadwal ujian.

Pilihan B tidak tepat karena penulis tidak menyatakan dukungannya terhadap protes


masyarakat tentang jadwal ujian.

Pilihan C dan D tidak tepat karena penulis tidak memasukkan perspektif atau


penilaian pribadinya terhadap protes masyarakat tentang jadwal ujian tersebut
sehingga tidak bisa dikatakan subjektif atau objektif. 

Jadi, pilihan jawaban yang tepat adalah E.

The question is based on the following picture.

75
SOAL-SOAL LITERASI BAHASA INGGRIS

93. Why did the boy thank to th e girl


a. Because she had answer his question
b. Because she had introduce herself to him
c. Because she had taken him to the principal’s room
d. Because she had showed the direction to the principal’s room.
e. Because she had invited to the principal’s room
Jawaban: A
Pembahasan: Sangat jelas

The question is based on the following text.

76
SOAL-SOAL LITERASI BAHASA INGGRIS

94. What does the author intend to show in his text?


A. That the quality of many sports shoes has greatly improved.
B. That it is best not to play football if you are under 12 years of age.

77
SOAL-SOAL LITERASI BAHASA INGGRIS

C. That young people are suffering more and more injuries due to their poor physical
condition.
D. That it is very important for young sports players to wear good sports shoes.
E. That the leather material is the best material for sport shoes due to its price.
Jawaban: D
Pembahasan: Jawaban terletak pada kalimat di atas gambar, mengenai seuatu
kecelakaan yang dialami oleh seorang atlit muda, yang secara tidak langsung
menyiratkan bahwa pentingnya bagi seorang atlit menggunakan sepatu yang bagus.

The following text is for number 95 – 96.

78
SOAL-SOAL LITERASI BAHASA INGGRIS

95. What is the purpose of the Key points?


A. To describe the dangers of using cell phones.
B. To suggest that debate about cell phone safety is ongoing.
C. To describe the precautions that people who use cell phones should take.
D. To suggest that there are no known health problems caused by cell phones.
E. To report the health problem caused by cell phones.
Jawaban: B

96. “It is difficult to prove that one thing has definitely caused another.”
What is the relationship of this piece of information to the Point 4 Yes and No
statements in the table Are cell phones dangerous?
A. It supports the Yes argument but does not prove it.
B. It proves the Yes argument.

79
SOAL-SOAL LITERASI BAHASA INGGRIS

C. It supports the No argument but does not prove it.


D. It shows that the No argument is wrong.
E. It supports neither the Yes nor the No arguments is true.
Jawaban: C

Read the passage and answer the question based on it.


A fact that draws our attention is that, according to his position in life, an extravagant man
is either admired or loathed. A successful business man does nothing to increase his
popularity by being prudent with his money. A person who is wealthy is expected to lead a
luxurious life and to be laviszh with his hospitality. If he is not so, he is considered mean, and
his reputation in business may even suffer in consequence. The paradox remains that he had
not been careful with his money in the first place; he would never have achieved his present
wealth.
Among the low income group, a different set of values exists. The young clerk, who
makes his wife a present of a new dress when he has not paid his house rent, is condemned as
extravagant. Carefulness with money to the point of meanness is applauded as a virtue.
Nothing in his life is considered more worthy than paying his bills. The ideal wife for such a
man separates her housekeeping money into joyless little piles – so much for rent, for food,
for the children’s shoes, she is able to face the milkman with equanimity every, month
satisfied with her economizing ways, and never knows the guilt of buying something she
can’t really afford.
As for myself, I fall neither of these categories. If I have money to spare I can be
extravagant, but when, as is usually the case, I am hard up and then I am the meanest man
imaginable.

97. Which of the following would be the most appropriate title for the passage:
A. Being extravagant is always condemnable.
B. The cause of poverty is extravagance.
C. Extravagance is a part of the rich as well as of the poor.
D. Stingy habits of the poor.
E. He would never have achieved his present wealth.
Jawaban: C

80
SOAL-SOAL LITERASI BAHASA INGGRIS

Please read the following passage below to answer the question.

Atmospheric pressure can support a column of water up to 10 meters high. But plants can
move water much higher, the sequoia tree can pump water to its very top, more than 100
meters above the ground. Until the end of the nineteenth century, the movement of water's in
trees and other talls plants was a mystery. Some botanists hypothesized that the living cells of
plants acted as pumps, but many experiments demonstrated that the stems of plants in which
all the cells are killed can still move water to appreciable heights. Other explanations for the
movement of water in plants have been based on root pressure, a push on the water from the
roots at the bottom of the plant. But root pressure is not nearly great enough to push water to
the tops of tall trees, Furthermore, the conifers, which are among the tallest trees have
unusually low root pressures.
If water is not pumped to the top of a tall tree, and if it is not pushed, to the top of a tall
tree, then we may ask. How does it get there? According to the currently accepted cohesion-
tension theory, water is pulled there. The pull on a rising column of water in a plant results
from the evaporation of water at the top of the plant. As water is lost from the surface of the
leaves, a negative pressure or tension is created. The evaporated water is replaced by water
moving from inside the plant in unbroken columns that extend from the top of a plant to its
roots. The same forces that create surface tension in any sample of water are responsible for
the maintenance of these unbroken columns of water. When water is confined in tubes of
very small bore, the forces of cohesion ( the attraction between water molecules) are so great
that the strength of a column of water compares with the strength of a steel wire of the same
diameter. This cohesive strength permits columns of water to be pulled to great heights
without being broken.

98. How do botanists know that root pressure is not the only force that moves water in
plants?
A. Some very tall trees have weak root pressure.
B. Root pressures decrease in winter.
C. Plants can live after their roots die.
D. Water in a plant's roots is not connected to water in its stem.
E. More than 100 meters above the ground

Jawaban: A

81
SOAL-SOAL LITERASI BAHASA INGGRIS

Please read the following passage below to answer the questions.

Fungi, of which there are over 100,000 species, including yeasts and other single-celled
organisms as well as the common molds and mushrooms, were formerly classified as
members of the plant kingdom. However, in reality they are very different from plants and
today they are placed in a separate group altogether. The principal reason for this is that none
of them possesses chlorophyll, and since they cannot synthesize their own carbohydrates,
they obtain their supplies either from the breakdown of dead organic matter or from other
living organisms. Furthermore the walls of fungal cells are not made of cellulose, as those of
plants are, but of another complex sugarlike polymer called chitin, the material from which
the hard outer skeletons of shrimps, spiders, and insects are made. The difference between the
chemical composition of the cell walls of fungi and those of plants is of enormous importance
because it enables the tips of the growing hyphae, the threadlike cells of the fungus, to secrete
enzymes that break down the walls of plant cells without having any effect on those of the
fungus itself. It is these cellulose-destroying enzymes that enable fungi to attack anything
made from wood, wood pulp, cotton, flax, or other plant material.

The destructive power of fungi is impressive. They are a major cause of structural damage
to building timbers, a cause of disease in animals and humans, and one of the greatest causes
of agricultural losses. Entire crops can be wiped out by fungal attacks both before and after
harvesting. Some fungi can grow at +50°C, while others can grow at -5°C, so even food in
cold storage may not be completely safe from them. On the other hand, fungi bring about the
decomposition of dead organic matter, thus enriching the soil and returning carbon dioxide to
the atmosphere. They also enter into a number of mutually beneficial relationships with
plants and other organisms. In addition, fungi are the source of many of the most potent
antibiotics used in clinical medicine, including penicillin.

99. Which of the following is mentioned as a major change in how scientists approach the
study of fungi?
A. Fungi are no longer classified as plants
B. Some single-cell organisms are no longer classified as fungi.
C. New methods of species identification have been introduced
D. Theories about the chemical composition of fungi have been revised.
E. They are very different from plants
Jawaban: A

82
SOAL-SOAL LITERASI BAHASA INGGRIS

100. According to the passage,how do fungi obtain carbohydrates?


A. The absorb carbohydrates from their own cell walls.
B. They synthesize chlorophyll to produce carbohydrates.
C. They produce carbohydrates by breaking down chitin.
D. They acquire carbohydrates from other organic matter, both living and dead.
E. The absorb carbohydrates from synthesize chlorophyll.
Jawaban: D

Please read the following passage below.

Certainly no creature in the sea is odder than the common sea cucumber. All living
creatureespecially human beings, have their peculiarities, but everything about the little sea
cucumber seems unusual. What else can be said about a bizarre animal that, among other
eccentricities, eats mud, feeds almost continuously day and night but can live without eating
for long periods, and can be poisonous but is considered supremely edible by gourmets?

For some fifty million years, despite all its eccentricities, the sea cucumber has subsisted on
its diet of mud. It is adaptable enough to live attached to rocks by its tube feet, under rocks in
shallow water, or on the surface of mud flats. Common in cool water on both Atlantic and
Pacific shores, it has the ability to such up mud or sand and digest whatever nutrients are
present.

Sea cucumbers come in a variety of colors, ranging from black to reddish-brown to sand-
color and nearly white. One form even has vivid purple tentacle. Usually the creatures are
cucumber-shapedhence their name-and because they are typically rock inhabitants, this
shape, combine with flexibility, enables them to squeeze into crevices where they are safe
from predators and ocean currents.

Although they have voracious appetites, eating day and night, sea cucumbers have the
capacity to become quiescent and live at a low metabolic rate-feeding sparingly or not at all
for long periods, so that the marine organisms that provide their food have a chance to
multiply. If it were not for this faculty, they would devour all the food available in a short
time and would probably starve themselves out of existence.

But the most spectacular thing about the sea cucumber is the way it defends itself. Its major
enemies are fish and crabs, when attacked, it squirts all its internal organs into the water. It

83
SOAL-SOAL LITERASI BAHASA INGGRIS

also casts off attached structures such as tentacles. The sea cucumber will eviscerate and
regenerate itself if it is attached or even touched; it will do the same if the surrounding water
temperature is too high or if the water becomes too polluted.

101. The fourth paragraph of the passage primarily discuss


A. the reproduction of sea cucumbers
B. the food sources of sea cucumbers
C. the shape of sea cucumbers is important
D. the eating habits of sea cucumbers
E. threats to sea cucumbers' existence

Jawaban: D

102. Of all the characteristics of the sea cucumber, which of the following seems to fascinate
the author most?
A. What it eats
B. Where it lives
C. What it does when threatened
D. How it hides from predators
E. How many characteristics of the sea cucumber

Jawaban: C

Read the passage and answer the question based on it.

Political education has many connotations. It may be defined as the preparation of a


citizen to take well informed, responsible and sustained action for participation in the national
struggle in order to achieve the socio-economic objectives of the country. The predominant
socio- economic objectives in India are the abolition of poverty and the creation of a modern
democratic, secular and socialist society in place of the present traditional, feudal,
hierarchical and in egalitarian one. Under the colonial rule, the Congress leaders argued that
political education was an important part of education and refused to accept the official view
that education and politics should not be mixed with one another. But when they came to
power in 1947 they almost adopted the British policy and began to talk of education being

84
SOAL-SOAL LITERASI BAHASA INGGRIS

defiled by politics. ‘Hands off education’ was the call to political parties. But in spite of it,
political infiltration into the educational system has greatly increased in the sense that
different political parties vie with each other to capture the mind of teachers and students.
The wise academicians wanted political support, without political interference. What we have
actually received is infinite political interference with little genuine political support. This
interference with the educational system by political parties for their own ulterior motives is
no political education at all and with the all round growth of elitism, it is hardly a matter for
surprise that real political education within the school system (which really means the
creation of a commitment to social transformation) has been even weaker than in the
preindependence period.
During that time only, the struggle for freedom came to an end and the major non-
formal agency of political education disappeared. The press played a major role by providing
some political education. But it did not utilize the opportunity to the full and the strangle hold
of vested interests continued to dominate it. The same can be said of political parties as well
as of other institutions and agencies outside the school system which can be expected to
provide political education. After analyzing all these things , it appears that we have made no
progress in genuine political education in the post-education period and have even slided
back in some respects. For instance, the education system has become even more elite-
oriented. Patriotism has become the first casualty. The father of the nation gave us the
courage to oppose government when it was wrong, in a disciplined fashion and on basic
principles. Today, we have even lost the courage to fight on basic issues in a disciplined
manner because agitational and anarchic politics for individual, group or party
aggrandizement has become common. In the recent times the education system continues to
support domination of the privileged groups and domestication of the underprivileged ones.
The situation will not change unless we take vigorous steps to provide genuine political
education on an adequate scale. This is one of the major educational reforms we need, and if
it is not carried out, mere linear expansion of the existing system of formal education will
only support the status quo and hamper radical social transformation.

103. According to the passage, what should be the main purpose of political education?
A. To champion the cause of elitism
B. To create an egalitarian society
C. To provide genuine political education
D. To bring qualitative change in the entire education system

85
SOAL-SOAL LITERASI BAHASA INGGRIS

E. To prepare the young generation with high intellectual acumen.


Jawaban: B

Read the passage and answer the question based on it.

Political education has many connotations. It may be defined as the preparation of a


citizen to take well informed, responsible and sustained action for participation in the national
struggle in order to achieve the socio-economic objectives of the country. The predominant
socio- economic objectives in India are the abolition of poverty and the creation of a modern
democratic, secular and socialist society in place of the present traditional, feudal,
hierarchical and in egalitarian one. Under the colonial rule, the Congress leaders argued that
political education was an important part of education and refused to accept the official view
that education and politics should not be mixed with one another. But when they came to
power in 1947 they almost adopted the British policy and began to talk of education being
defiled by politics. ‘Hands off education’ was the call to political parties. But in spite of it,
political infiltration into the educational system has greatly increased in the sense that
different political parties vie with each other to capture the mind of teachers and students.
The wise academicians wanted political support, without political interference. What we have
actually received is infinite political interference with little genuine political support. This
interference with the educational system by political parties for their own ulterior motives is
no political education at all and with the all round growth of elitism, it is hardly a matter for
surprise that real political education within the school system (which really means the
creation of a commitment to social transformation) has been even weaker than in the
preindependence period.

During that time only, the struggle for freedom came to an end and the major non- formal
agency of political education disappeared. The press played a major role by providing some
political education. But it did not utilize the opportunity to the full and the strangle hold of
vested interests continued to dominate it. The same can be said of political parties as well as
of other institutions and agencies outside the school system which can be expected to provide
political education. After analyzing all these things , it appears that we have made no
progress in genuine political education in the post-education period and have even slided
back in some respects. For instance, the education system has become even more elite-
oriented. Patriotism has become the first casualty. The father of the nation gave us the

86
SOAL-SOAL LITERASI BAHASA INGGRIS

courage to oppose government when it was wrong, in a disciplined fashion and on basic
principles. Today, we have even lost the courage to fight on basic issues in a disciplined
manner because agitational and anarchic politics for individual, group or party
aggrandizement has become common. In the recent times the education system continues to
support domination of the privileged groups and domestication of the underprivileged ones.
The situation will not change unless we take vigorous steps to provide genuine political
education on an adequate scale. This is one of the major educational reforms we need, and if
it is not carried out, mere linear expansion of the existing system of formal education will
only support the status quo and hamper radical social transformation.

104. How has politics been related to educational institutions after independence?
A. They got political support at the cost of political interference.
B. There was substantial interference without political support.
C. Although they got political support but there was no interference of politics.
D. The education system mainly represents the oppressed sections of the society.
E. It is clear that they got almost no political support as well as political interference.
Jawaban: B

Read the passage and answer the question based on it.

If a person suddenly encounters any terrible danger, the change of nature one undergoes is
equally great. Sometimes fear numbs our senses. Like animals, one stands still, powerless to
move a step in fright or to lift a hand in defense of our lives, and sometimes one is seized
with panic, and again, act more like the inferior animals than rational beings. On the other
hand, frequently in cases of sudden extreme peril, which cannot be escaped by flight, and
must be instantly faced, even the most timid men at once as if by miracle, become possessed
of the necessary courage, sharp quick apprehension and swift decision. This is a miracle very
common in nature. Man and the inferior animals alike, when confronted with almost certain
death ‘ gather resolution from despair’ but there can really be no trace of so debilitating a
feeling in the person fighting, or prepared to fight for dear life. At such times the mind is
clearer than it has ever been; the nerves are steel, there is nothing felt but a wonderful
strength and daring. Looking back at certain perilous moments in my own life, I remember
them with a kind of joy, not that there was any joyful excitement then, but because they
broadened my horizon, lifted me for a time above myself.

87
SOAL-SOAL LITERASI BAHASA INGGRIS

105. The title that best suits the passage would be:
A. The Will to Fight
B. The Miracle of Confronting Danger
C. The Change of Nature
D. Courage and Panic
E. A wonderful strength and daring
Jawaban: B

Read the passage and answer the question based on it.

Caffeine, the stimulant in coffee, has been called “the most widely used psychoactive
substance on Earth.” Synder, Daly and Bruns have recently proposed that caffeine affects
behavior by countering the activity in the human brain of a naturally occurring chemical
called adenosine. Adenosine normally depresses neuron firing in many areas of the brain. It
apparently does this by inhibiting the release of neurotransmitters, chemicals that carry nerve
impulses from one neuron to the next. Like many other agents that affect neuron firing,
adenosine must first bind to specific receptors on neuronal membranes. There are at least two
classes of these receptors, which have been designated A1 and A2.

Snyder et al propose that caffeine, which is structurally similar to adenosine, is able to


bind to both types of receptors, which prevents adenosine from attaching there and allows the
neurons to fire more readily than they otherwise would. For many years, caffeine’s effects
have been attributed to its inhibition of the production of phosphodiesterase, an enzyme that
breaks down the chemical called cyclic AMP. A number of neurotransmitters exert their
effects by first increasing cyclic AMP concentrations in target neurons. Therefore, prolonged
periods at the elevated concentrations, as might be brought about by a phosphodiesterase
inhibitor, could lead to a greater amount of neuron firing and, consequently, to behavioral
stimulation. But Snyder et al point out that the caffeine concentrations needed to inhibit the
production of phosphodiesterase in the brain are much higher than those that produce
stimulation. Moreover, other compounds that block phosphodiesterase’s activity are not
stimulants.

88
SOAL-SOAL LITERASI BAHASA INGGRIS

To buttress their case that caffeine acts instead by preventing adenosine binding, Snyder
et al compared the stimulatory effects of a series of caffeine derivatives with their ability to
dislodge adenosine from its receptors in the brains of mice. “In general,” they reported, “the
ability of the compounds to compete at the receptors correlates with their ability to stimulate
locomotion in the mouse; i.e., the higher their capacity to bind at the receptors, the higher
their ability to stimulate locomotion.” Theophylline, a close structural relative of caffeine and
the major stimulant in tea, was one of the most effective compounds in both regards. There
were some apparent exceptions to the general correlation observed between adenosine-
receptor binding and stimulation. One of these was a compound called 3-isobuty1-1-
methylxanthine(IBMX), which bound very well but actually depressed mouse locomotion.
Snyder et al suggest that this is not a major stumbling block to their hypothesis. The problem
is that the compound has mixed effects in the brain, a not unusual occurrence with
psychoactive drugs. Even caffeine, which is generally known only for its stimulatory effects,
displays this property, depressing mouse locomotion at very low concentrations and
stimulating it at higher ones.

106. The primary purpose of the passage is to


A. discuss a plan for investigation of a phenomenon that is not yet fully understood
B. present two explanations of a phenomenon and reconcile the differences between
them
C. summarize two theories and suggest a third theory that overcomes the problems
encountered in the first two
D. describe an alternative hypothesis and provide evidence and arguments that
support it
E. challenge the validity of a theory by exposing the inconsistencies and
contradictions in it
Jawaban: D

Read the passage and answer the question based on it.

Many United States companies have, unfortunately, made the search for legal protection
from import competition into a major line of work. Since 1980 the United States International
Trade Commission (ITC) has received about 280 complaints alleging damage from imports

89
SOAL-SOAL LITERASI BAHASA INGGRIS

that benefit from subsidies by foreign governments. Another 340 charge that foreign
companies “dumped” their products in the United States at “less than fair value.” Even when
no unfair practices are alleged, the simple claim that an industry has been injured by imports
is sufficient grounds to seek relief.

Contrary to the general impression, this quest for import relief has hurt more companies
than it has helped. As corporations begin to function globally, they develop an intricate web
of marketing, production, and research relationships, The complexity of these relationships
makes it unlikely that a system of import relief laws will meet the strategic needs of all the
units under the same parent company.

Internationalization increases the danger that foreign companies will use import relief laws
against the very companies the laws were designed to protect. Suppose a United States-
owned company establishes an overseas plant to manufacture a product while its competitor
makes the same product in the United States. If the competitor can prove injury from the
imports—and that the United States company received a subsidy from a foreign government
to build its plant abroad—the United States company’s products will be uncompetitive in the
United States, since they would be subject to duties.

Perhaps the most brazen case occurred when the ITC investigated allegations that
Canadian companies were injuring the United States salt industry by dumping rock salt, used
to de-ice roads. The bizarre aspect of the complaint was that a foreign conglomerate with
United States operations was crying for help against a United States company with foreign
operations. The “United States” company claiming injury was a subsidiary of a Dutch
conglomerate, while the “Canadian” companies included a subsidiary of a Chicago firm that
was the second-largest domestic producer of rock salt

107. It can be inferred from the passage that the minimal basis for a complaint to the
International Trade Commission is which of the following?
A. A foreign competitor has received a subsidy from a foreign government.
B. A foreign competitor has substantially increased the volume of products shipped
to the United States.
C. A foreign competitor is selling products in the United States at less than fair
market value.

90
SOAL-SOAL LITERASI BAHASA INGGRIS

D. The company requesting import relief has been injured by the sale of imports in
the United States.
E. The company requesting import relief has been barred from exporting products to
the country of its foreign competitor.
Jawaban: D

Please read the following passage below to answer the question.

For the agent, however, reason is the heart of the matter. And the heart of the matter is
the reason for the civitas, whether it is vocalized or not. What is at stake in what I do is the
kind of person I become. What is at stake in what we do is the kind of city we inhabit. In both
the individual and the social variation of that mantra, familiar from virtue ethics, every action
is the conclusion of a practical syllogism; it carries with it an argument, and the argument
underwrites both character and civitas. The relationship is circular: character forms and is
formed by every action, and each action tends to confirm the character of the agent. And the
city forms and is formed by the characters it contains. When Paine says that the long habit of
not thinking a thing wrong creates the superficial impression of its being right, he brings to
our attention the fact that the long habit of not thinking a thing wrong makes it unlikely that
we will think to change it. The shape of the city, like the shape of character, is a
counterweight to change. On the positive side, this makes cities and characters relatively
stable; and it gives us some idea what to expect of them if we have been paying attention. On
the negative side, this renders characters and cities largely impervious to reason. Time makes
more converts than reason, but time also tends, for better or worse, to confirm reasons of the
heart that reason cannot know. What passes as stability may simply be inertia.
Revolutionary theory turns on this question: is it stability or is it inertia? Either way,
change is–and should be–difficult. For conservative theorists such as Burke, this translates
into gradualism. The civitas changes in the manner of an organism, maturing in time and
evolving across generations. Sudden change is the exception, not the rule. And, to a large
extent, revolutionary theorists agree. Jefferson felt obliged to document a long pattern of
abuse as justification for a single violent act. David Walker, writing with Jefferson in mind
and partly in response to his Notes on the State of Virginia, followed the same pattern.
Thoreau urged readers to let the ordinary friction of civil society pass and reserve
disobedience for consistent affronts to human dignity. The African National Congress
documented centuries of abuse before turning to armed resistance. Jefferson and Mao Zedong

91
SOAL-SOAL LITERASI BAHASA INGGRIS

both asserted that every generation needed its own revolution, but, even so, they agreed that
every revolution required reason. The whole world, for Jefferson, is a court before which the
revolutionary has to make a case. That the New Left in the United States took up this
Jeffersonian approach is reflected in its most simplified form by the chant “the whole world
is watching” that framed demonstrations in Chicago in 1968. That the actions were (and are)
called demonstrations suggests, at least, an audience and something to be demonstrated. Both
revolution and war are put forward as rhetorical strategies within an argument that involves
the world as a whole. Drawing the whole world into every act of violence may partly explain
why “local” wars and revolutions have escalated into global conflicts. But my point here is to
focus on rhetorical strategy in the context of an argument. Thoreau was convinced that no act
was rhetorically insignificant, and both Gandhi and King followed him in this. The most
revolutionary act in Gandhi’s account was spinning the thread with which to make the clothes
one wore. And this is critical to civil disobedience as a rhetorical strategy.
One common thread in rational justifications for war and revolution is the documentation
of violence and abuse against which war or revolution is a reaction. War and revolution are
invariably depicted as last resorts: they are justified when there is nothing else to be done.
This is hardly surprising, since the strongest argument demonstrates a necessary conclusion.
If the conclusion is necessary, then disagreement with it is nonsensical.

108. With which of the following argument is the author non committal in the entire
passage?
A. Sudden change is not a law but an exemption
B. Local wars and revolutions escalating into global conflicts.
C. Perpetual revolution is not a conclusion but a step in the argument
D. War and revolution are justified when there is nothing else to be done
E. Revolution and war are put forward as rhetorical strategies within an argument
Jawaban: B

The question is based on the following text.

(The story by Oiwan Lam originally appeared on Global Voices, on December 2, 2020)
    Over a dozen journalists resigned on December 1 from Hong Kong Cable Television
Limited, a major subscription-based TV station in Hong Kong, in protest after the media

92
SOAL-SOAL LITERASI BAHASA INGGRIS

company fired 40 editorial staff in what is perceived as a political crackdown. Those laid off
include the head of the China desk and the entire team of the award-winning News Lancet
program. The broadcaster cited financial reasons for the layoff, a claim the newsroom staff
rebukes. The entire China desk, a total of 12 people, has since resigned in protest, along with
the heads of the international and finance desks and the newsroom's chief assignment editor.

    The layoffs were sudden and took immediate effect — those who were sacked were told to
leave the office on the same day. In shock, members of staff gathered outside the room of the
news director demanding an explanation. Among those laid off is Wong Lai-ping, former
deputy news editor at the China desk. On a radio interview aired on December 2, she rebuked
the management's claim that the decision was made out of financial difficulties, remarking
that in August, the management fired the then-executive director Fung Tak-hung and
replaced him with four new managers. Wong said that the new management often made
editorial requests, such as live broadcasts of China's foreign minister's regular press
conference, or modifying headlines. According to Wong, the newsroom staff refused to
comply with such requests.

    The Hong Kong Journalists Association commented on the layoffs, “Given the [New
Lancet] team’s coverage of the police and administration, it’s difficult not to see this as
minimizing sensitive reporting in the name of cost-cutting”.

“..., she rebuked the management's claim that the decision was made out of financial
difficulties, remarking that in August, the management fired the then-executive director Fung
Tak-hung and replaced him with four new managers.” (Paragraph 2).

109. Dealing with the issue discussed in the text, the statement above may lead the readers to
the assumption that …
A. The management’s claim should have been accepted by all parties.
B. The decision to fire Fung Tak-hung was taken with well-consideration.
C. The director Fung Tak-hung was not as competent as four new managers.
D. There was an illogical situation to claim financial difficulties as the reason for the
layoffs.
E. The management tried to cover up their financial difficulties by cutting the
number of employees.

93
SOAL-SOAL LITERASI BAHASA INGGRIS

Jawaban: D

Pembahasan:

Arti pernyataan di atas adalah “..., dia menegur klaim manajemen bahwa keputusan
itu dibuat karena kesulitan keuangan, dengan mengatakan bahwa pada bulan
Agustus, manajemen memecat direktur Fung Tak-hung dan menggantinya dengan
empat manajer baru.”. Pada soal ini, kita harus menentukan asumsi yang mungkin
timbul dari pernyataan tersebut. 

Teks di atas menyampaikan sebuah berita mengenai pengunduran diri lebih dari 12
jurnalis sebagai bentuk protes mereka terhadap pemecatan 40 staf editorial yang
diklaim atas tindak lanjut dari kesulitan keuangan yang dialami perusahaan
tersebut. Namun, pernyataan yang terdapat pada soal ini menyebutkan bahwa
manajemen justru merekrut empat manajer baru setelah memecat seorang direktur.
Pernyataan tersebut menunjukkan bahwa perusahaan tersebut terkesan “masih
memiliki cukup uang” karena mereka merekrut 4 orang manajer sekaligus padahal
mereka sebelumnya memecat satu orang dengan alasan kesulitan keuangan.
Logikanya, jika perusahaan tersebut sedang mengalami kesulitan keuangan,
mereka akan berpikir lagi untuk merekrut mereka sekaligus. Dengan demikian,
asumsi yang paling mungkin timbul dari pernyataan tersebut adalah (D) “Terdapat
situasi tidak logis untuk mengklaim kesulitan keuangan sebagai alasan
dari layoffs atau PHK.”.

Jadi, pilihan jawaban yang tepat adalah D.

The questions no. 110 – 111 are based on the following passage.

(The story by Sanjib Chaudhary originally appeared on Global Voices on August 8, 2020)

    After news of the existence of a yellow turtle in India's eastern state of Odisha went viral
this past July, Nepalis wanted to remind the world that they made a similar discovery first —
on April 14, 2018, to be exact — when a rare, golden turtle  — but a different species — was
found for the first time in southeastern Nepal’s Dhanushadham municipality. According to a
research paper by Kamal Devkota, Dev Narayan Mandal and Hinrich Kaiser, which was

94
SOAL-SOAL LITERASI BAHASA INGGRIS

published in the journal Herpetology Notes, the turtle was released into its natural habitat
after pictures were taken as proof. 

    A normal Indian flapshell turtle is greenish-grey in colour, with yellow marks on its head
and neck. It has a grey carapace dotted with dark yellow spots and derives its name from the
femoral flaps at the plastron, the ventral part of the shell. These turtles are found in
Bangladesh, India, Myanmar, Nepal, Pakistan and Sri Lanka, most typically in ditches, lakes,
ponds, and paddy fields with stagnant water. Omnivorous in nature, they eat anything from
leaves and flowers to snails, fish, and frogs. While a normal-coloured flapshell turtle can
easily camouflage itself in the murky, greenish water, its golden-coloured variant — a turtle
with chromatic leucism — is easily recognisable and therefore more vulnerable. Its luminous
golden colour, in particular, makes it a prized pet.

    The congenital disorder of albinism creates a complete absence of pigmentation in the skin,
hair, and eyes due to the lack of tyrosinase, an enzyme involved in the production of melanin.
Researchers estimate that albinism occurs once in every 10,000 mammal births. Leucism, on
the other hand, is an extremely rare genetic condition in which animals have reduced
pigmentation. Most leucistic animals have normal-coloured eyes, whereas those with
albinism tend to have red or pink eyes.

    Nepal is home to 16 species of turtles, of which four are critically endangered: the three-
striped roofed turtle (Kachuga dhongoka), the red-crowned roofed turtle (Kachuga kachuga),
the Indian narrow-headed softshell turtle (Chitra indica), and the elongated tortoise
(Indotestudo elongata). Though turtles play an important role in reducing pollution by
feeding on insects, vegetation, and dead animals, factors like habitat loss, fragmentation, and
degradation have been threatening their survival. According to “Turtles of Nepal – A Field
Guide for Species Accounts and Distribution,” the draining of wetlands for irrigation and
fishing, the excessive use of agrochemicals which eventually seep into waterways, and
unsustainable fishing methods all negatively impact turtle populations.

110. Which of the following is NOT TRUE according to the passage above?
A. A quarter of all turtle species in Nepal are threatened with extinction.
B. Both Nepalis and Indians found new rare turtles at different times.
C. A turtle with chromatic leucism is insusceptible topredators.
D. Animals with albinism and leucism can bedistinguished.

95
SOAL-SOAL LITERASI BAHASA INGGRIS

E. Turtles can help humans to keep the environment healthy.


Jawaban: C

Pembahasan:

Soal menanyakan pernyataan yang tidak tepat berdasarkan teks.

Pilihan A (Seperempat dari seluruh spesies penyu di Nepal terancam


punah) sesuai dengan pernyataan “Nepal is home to  16 species of turtles, of
which  four  are critically endangered: ....” (Nepal adalah rumah bagi 16 spesies
penyu, yang 4 diantaranya terancam punah: ....). Empat dari 16 spesies sama artinya
dengan seperempat-nya.

Pilihan B (Orang Nepal dan India menemukan penyu langka baru pada waktu yang
berbeda) sesuai dengan pernyataan “After news of the existence of a yellow turtle
in India's eastern state of Odisha went viral this past July, Nepalis wanted to
remind the world that they made a similar discovery first — on April 14,
2018, ...”. Bagian yang dicetak tebal menunjukkan bahwa orang Nepal menemukan
penyu langka sebelum orang India menemukan spesies yang serupa. Dengan
demikian, orang Nepal dan India menemukan penyu langka baru pada waktu
yang berbeda.

Pilihan C (Seekor penyu dengan chromatic leucism tidak rentan menjadi


mangsa) tidak sesuai atau bertentangan dengan pernyataan “... its golden-
coloured variant — a turtle with chromatic leucism — is easily recognisable and
therefore more vulnerable.” (... variannya yang berwarna keemasan, penyu
dengan chromatic leucism mudah dikenali dan karena itulah lebih rentan/mudah
diserang.). Keadaan mudah diserang ini menyiratkan bahwa penyu tersebut juga
akan mudah dijadikan mangsa oleh predator.

Pilihan D (Hewan dengan albinisme dan leucisme dapat dibedakan) sesuai dengan


pernyataan “Most leucistic animals have normal-coloured eyes, whereas those with
albinism tend to have red or pink eyes.”. Kalimat tersebut menunjukkan bahwa
mereka dapat dibedakan dari warna matanya.

Pilihan E (Penyu dapat membantu manusia untuk menjaga lingkungan tetap

96
SOAL-SOAL LITERASI BAHASA INGGRIS

sehat) sesuai dengan pernyataan “Though turtles  play an important role in


reducing pollution  ….”. Perannya “mengurangi polusi” merupakan salah satu faktor
yang dapat membuat lingkungan kita tetap sehat (tanpa adanya polusi).

Jadi, pernyataan yang tidak tepat adalah C.

111. The paragraph following the passage will most likely discuss .…
A. research on all turtle populations
B. dangerous factors for turtle populations
C. the suggestion to look after our environment
D. turtle populations’ contribution to reduce pollution
E. some ways to save turtles from threatening factors
Jawaban: E

Pembahasan:

Soal menanyakan topik yang mungkin dibahas setelah paragraf terakhir. Untuk
menjawab soal seperti ini, fokuskan pada paragraf terakhir, khususnya kalimat
terakhir.

Arti kalimat terakhir adalah “Menurut Turtles of Nepal - A Field Guide for Species
Accounts and Distribution, pengeringan lahan basah untuk irigasi dan penangkapan
ikan, penggunaan bahan kimia berlebihan yang akhirnya meresap ke saluran air, dan
metode penangkapan ikan yang tidak berkelanjutan berdampak negatif pada
populasi penyu.". Pada kalimat sebelumnya, penulis membahas kelangsungan
hidup penyu yang terancam karena beberapa faktor padahal mereka berperan
penting dalam pengurangan polusi.

Berdasarkan hal tersebut, paragraf selanjutnya kemungkinan besar akan membahas


“cara menyelamatkan penyu dari faktor-faktor yang mengancam kelangsungan hidup
mereka tersebut”.

Pilihan A (penelitian terhadap semua populasi penyu) tidak tepat karena seharusnya


penelitian seperti ini disampaikan sebelumnya, khususnya sebelum menyatakan

97
SOAL-SOAL LITERASI BAHASA INGGRIS

bahwa populasi mereka terancam.

Pilihan B (faktor yang berbahaya bagi populasi penyu) dan D (kontribusi populasi


penyu berkontribusi untuk mengurangi polusi) tidak tepat karena justru hal ini telah
dijelaskan pada paragraf terakhir teks tersebut.

Pilihan C (saran untuk menjaga lingkungan kita) tidak tepat karena seharusnya


saran tersebut lebih dikhususkan pada menjaga populasi penyunya.
Jadi, pilihan jawaban yang tepat adalah E.

Questions 112 to 114 are based on the following text.

The latest round in an ongoing debate over global-warming trends claims that warming has
indeed slowed down this century. An obvious slowing in the rise of global temperatures was
recorded at the beginning of the twenty-first century. This was referred to as a "hiatus" or a
"pause". This hiatus was first observed several years ago. Climate-change skeptics have used
this as evidence that global warming has stopped permanently. But in June the previous year,
a study in science claimed that the hiatus was just an artifact which disappears when biases in
temperature data are corrected.

Now a prominent group of researchers is countering that claim. They argue in Nature Climate
Change that even after correcting these biases the slowdown was real. "There is this
mismatch between what the climate models are producing and what the observations are
showing," says lead author John Fyfe. "We can't ignore it." Fyfe uses the term "slowdown"
rather than "hiatus". He also stresses that it does not in any way weaken global-warming
theory.

The study that questioned the existence of the slowdown corrected known biases in the
surface temperature record maintained by the US National Oceanic and Atmospheric
Administration (NOAA). The finding showed differences in temperature readings from ships
and buoys. This effectively increased the record about warming. The researchers also
extended the record to include 2014. This set a new record high for average temperatures.

98
SOAL-SOAL LITERASI BAHASA INGGRIS

Thomas Karl, director of National Centers for Environmental Information in Asheville,


calculated the rate of global warming between 1950 and 1999 as being 0.113°C per decade.
This was similar to the 0.116°C a decade calculated for 2000-14. This, Karl said, meant that
an assessment done by the influential Intergovernmental Panel on Climate Change in 2013
showing that warming had slowed was no longer valid.

112. The passage above mainly discusses about….


A. the hiatus observation was first carried out several years ago
B. the reason why global warming is slowing down in this century
C. global warming is a verifiable issue in the space of a decade
D. the contention about global warming and whether it is indeed slowing this period
E. the view of study in science that the hiatus is an artifact which vanishes
Jawaban: D

Pembahasan:
Perhatikan kalimat pertama pada paragraf ke-1 “The latest round in an ongoing
debate over global-warming trends claims that warming has indeed slowed down
this century.” inti kalimatnya mengatakan adanya perdebatan yang membahas
tentang tren pemanasan global yang melambat abad ini. Maka topik dari teks di atas
adalah (D) The contention about global warming and whether it is indeed slowing
this period.

113. The word “prominent” in paragraph 2 means….


A. well-known
B. promenade
C. shrewd
D. indolent
E. magnificent

Jawaban: A
Pembahasan:
Prominent (Terkenal) memiliki makna yang sama dengan pilihan jawaban (A) well-
known. Sedangkan, pilihan jawaban (B) promenade artinya “perjalanan”,
(C) shrewd berarti “cerdas”, (D) indolent memiliki arti “malas”, dan

99
SOAL-SOAL LITERASI BAHASA INGGRIS

(E) magnificent  artinya “sangat bagus”.

114. Why have some claimed that global warming a fabricated issue?
A. Because there’s no valid data to prove that global warming is real.
B. Since the existence of the slowdown corrected known biases in the surface
temperature record upheld by the US NOAA.
C. As an assessment done on Climate Change presenting that warming had slowed
was no longer valid.
D. For the researchers is countering that in Nature Climate Change even after
correcting these biases the slowdown was real.
E. Because of the diversification in temperature readings from ships and buoys, the
study found.

Jawaban: C
Pembahaan:
Pilihan jawaban (C) merupakan opsi yang tepat karena pada paragraf ke-4 “This,
Karl said, meant that an assessment done by the influential Intergovernmental Panel
on Climate Change in 2013 showing that warming had slowed was no longer
valid.” dikatakan bahwa perhitungan yang dilakukan oleh Panel Antarpemerintah
pada Perubahan Iklim menunjukkan bahwa pemanasan telah melambat tidak lagi
berlaku, yang mana artinya bahwa pemanasan global merupakan isu yang tidak
benar.

The following text is for questions 115 to 116.

Over the last two decades, the use of ICT has been an important topic in education. On the
one hand, studies have shown that ICT can enhance teaching and learning outcomes. For
example, in science and mathematics education, scholars have documented that the use of
ICT can improve students’ conceptual understanding, problem solving, and team working
skills. Consequently, most curriculum documents state the importance of ICT and encourage
school teachers to use them. However, teachers need to specifically trained in order to
integrate ICT in their teaching.

100
SOAL-SOAL LITERASI BAHASA INGGRIS

Schools are known to be resistant to innovation and change, however, the spread of ICT is
beginning to affect how teachers teach. One of the current issues about the use of ICT is how
it is integrated into the curriculum. The curriculum document provide arguments for
introducing ICT in the school setting. Therefore, schools expect that graduates from teacher
education programs have a reasonable knowledge of how to use ICT. However, this may not
be the case because most current teachers’ pre-service preparation, and subsequent in-service
courses were designed by using traditional educational technology and settings. Thus, the
participants in these courses are not familiar with the processes, interaction patterns, features,
and possibilities of teaching learning processes based on ICT.

Effective development of pre-service teachers’ ICT proficiency does not seem to be a direct
process, but is the one asking for a careful, complex approach. First, a need assessment is
important to find out what ICT skills and knowledge teachers need at schools. Second,
designers of teacher education programs should know the pre-service teachers’ perceptions of
ICT and their attitudes toward ICT integration into curriculum. Third, teacher education
programs need to consider the two typical arguments that support the ICT use in schools.

115. Which of the following best restates the sentence “Over the last two decades, the use of
ICT has been an important topic in education. On the one hand, studies have shown that
ICT can enhance teaching and learning outcomes.” in paragraph 1?
A. ICT usage has been a vital topic in education and studies indicate that ICT can
develop teaching and learning upshot.
B. Studies have shown that ICT can enrich teaching and learning end result
C. The use of ICT has been an insignificant topic in education. Also, studies have
shown that ICT can enhance teaching and learning outcomes.
D. Over the last two decades, the use of ICT has been an important topic in
educational method.
E. Over the preceding two decades, studies have exposed that ICT can increase
teaching and learning outcomes.
Jawaban: A

Pembahasan:
Dalam pernyataan “Selama dua dekade terakhir, penggunaan ICT telah menjadi topik
penting dalam pendidikan. Di satu sisi, penelitian telah menunjukkan bahwa TIK

101
SOAL-SOAL LITERASI BAHASA INGGRIS

dapat meningkatkan hasil pengajaran dan pembelajaran.” dapat dinyatakan kembali


oleh opsi (A) ICT usage has been a vital topic in education and studies indicate that
ICT can develop teaching and learning upshot.

116. Based on the passage, paragraph 1 most likely discusses….


A. educational growth in the last two decades
B. the development of education by using ICT
C. the advantages of using ICT in the learning process
D. teachers need to be trained in order to integrate ICT in their teaching
E. ICT and education in today’s learning development

Jawaban: B
Pembahasan:
Pertanyaan soal ini intinya menanyakan topik paragraf ke-1. Paragraf ke-1 topiknya
membahas penggunaan TIK telah menunjukkan bahwa TIK dapat meningkatkan
hasil pengajaran dan pembelajaran “On the one hand, studies have shown that ICT
can enhance teaching and learning outcomes.”  itu artinya adanya pengembangan
pendidikan dengan menggunakan TIK. Maka pilihan jawaban (B) the development of
education by using ICT merupakan opsi yang tepat.

Questions 117 - 120 are based on the following passage.

How are we able to find things in the dark? It is because our brain is able to store information
in such a way that it can be retrieved by different senses. It turns out that the ability to
recognise objects across different senses is present in the tiny brains of an insect. Researchers
at Queen Mary University of London and Macquarie University in Sydney have published
new work in the journal Science showing that bumblebees can also find objects in the dark
they've only seen before.

In the light, but barred from touching the objects, bumblebees were trained to find rewarding
sugar water in one type of object (cubes or spheres) and bitter quinine solution in the other
shape. When tested in the dark, bees preferred the object that was previously rewarding,
spending more time exploring them.

102
SOAL-SOAL LITERASI BAHASA INGGRIS

Dr Cwyn Solvi is the lead author on the paper who was based at Queen Mary University of
London and is now at Macquarie University in Sydney. She said: "The results of our study
show that bumblebees don't process their senses as separate channels -- they come together as
some sort of unified representation."

Selene Gutierrez Al-Khudhairy, co-author on the paper, and now PhD student at the
University of York, said: "This is an amazing feat when you consider the miniscule size of a
bee's brain. Future investigations of the neural circuitry underlying this ability in bees may
one day help reveal how our own brains imagine the world as we do."

117. What is the writer’s purpose in writing the passage?


A. To explain how bumblebees react to the light.
B. To motivate the readers to preserve bumblebees in the wild.
C. To reveal how important bumblebees are to human survival.
D. To inform a study on the ability of bumblebees in the absence of light.
E. To report the procedure of a study on bumblebees conducted by Dr Cwyn Solvi.
Jawaban : D

Pembahasan: 

Pada paragraf pertama, penulis menjelaskan tentang temuan bahwa lebah dapat
mengenali benda yang mereka lihat sebelumnya, dalam gelap. Paragraf kedua
membahas percobaan yang melibatkan aktivitas lebah dalam situasi terang dan gelap.
Paragraf ketiga dan keempat membahas tentang pendapat para ahli mengenai hasil
penelitian tentang kemampuan melihat lebah dalam gelap Jadi, tujuan penulis
menuliskan teks tersebut adalah D. Untuk menginformasikan penelitian tentang
kemampuan lebah tanpa adanya cahaya.

118. The word “barred” in paragraph two can be best replaced by….
A. blocked
B. distracted
C. permitted
D. provided
E. unsealed
Jawaban : A

103
SOAL-SOAL LITERASI BAHASA INGGRIS

Pembahasan :
“In the light, but barred from touching the objects” menunjukkan bahwa penelitian
terhadap lebah tersebut dilakukan dalam terang, tetapi si lebah terhalang dari
menyentuh objek yang ia lihat. Sinonim dari kata “barred” yang artinya terhalang
adalah “blocked” sehingga jawaban yang tepat adalah A.

119. In which paragraph is it mentioned about the benefits of the research for humans?
A. 1
B. 2
C. 3
D. 4
E. 5
Jawaban: D
Pembahasan:
Informasi mengenai manfaat penelitian bagi manusia dapat ditemui pada paragraf
keempat: “Future investigations of the neural circuitry underlying this ability in
bees may one day help reveal how our own brains imagine the world as we do.”
yang artinya penelitian lebih lanjut tentang sirkuit saraf yang mendasari kemampuan
ini (kemampuan “melihat” dalam gelap pada lebah) dapat suatu saat mengungkap
bagaimana otak kita membayangkan dunia.

120. The writer’s intention in writing the sentence “It turns out that the ability to recognise
objects across different senses is present in the tiny brains of an insect.” in paragraph 1
is to…
A. confirm that humans and insects are quite similar.
B. emphasize the need for a research on certain insects.
C. prove that bumblebees can see in the dark as well as humans.
D. exemplify the kinds of insects which are able to sense objects in the dark.
E. pinpoint that humans are not the only one who possess the ability mentioned.
Jawaban : E
Pembahasan:
Pada kalimat tersebut, penulis secara tersirat menyatakan bahwa kemampuan untuk
“melihat” dalam gelap ternyata bukanlah sesuatu yang hanya dimiliki oleh manusia.

104
SOAL-SOAL LITERASI BAHASA INGGRIS

Ada makhluk lain yang juga memiliki kemampuan tersebut, yaitu lebah.

105

You might also like